Está en la página 1de 28

1

I. The Public Land Act (Commonwealth Act 141) 1. The Republic of the Philippines should have exhausted all fully measuring the Court's discretion, enumerates the
Preliminaries administrative remedies before filing the case in court; premises for granting a review:

G.R. No. L-50464 January 29, 1990 2. The title issued to SUNBEAM and CORAL BEACH had (a) When the Court of Appeals has decided a question of
SUNBEAM CONVENIENCE FOODS INC., CORAL BEACH become indefeasible and imprescriptible; substance, not theretofore determined by the Supreme Court
DEVELOPMENT CORP., and the REGISTER OF DEEDS OF or has decided it in a way probably not in accord with law or
BATAAN, petitioners, vs. HON. COURT OF APPEALS and 3. The action for reversion was defective, having been initiated the applicable decisions of the Supreme Court; and
THE REPUBLIC OF THE PHILIPPINES, respondents. by the Solicitor General and not by the Director of Lands. 3
SARMIENTO, J.: (b) When the Court of Appeals has so far departed from the
The then Court of First Instance of Bataan dismissed the accepted and usual course of judicial proceedings or so far
In this petition for review on certiorari, Convenience Foods complaint in the Order of October 7, 1977,4 adopting mainly sanctioned such departure by a lower court as to call for
Corporation (hereafter simply SUNBEAM) and Coral Beach the theory that since the titles sought to be cancelled supervision .7
Development Corporation (hereafter simply CORAL BEACH) emanated from the administrative act of the Bureau of Lands
bring to our attention the decision rendered by the Court of Director, the latter, not the courts, had jurisdiction over the We agree with the Court of Appeals' granting of the petition
Appeals in "Republic of the Philippines v. Hon. Pedro T. disposition of the land. filed by the Republic of the Philippines charging the then Court
Santiago, et al.," disposing as follows: of First Instance with grave abuse of discretion. The filing of
The Solicitor General received the copy of the Order on the Motion to Dismiss the complaint for reversion by
October 11, 1977 and filed a Notice of Appeal dated October SUNBEAM and CORAL BEACH on the ground of lack of
25, 1977 .5 The Solicitor General then moved for an extension cause of action, necessarily carried with it the admission, for
WHEREFORE, the writ prayed for is granted. The order of the of thirty days within which to file the Record on Appeal and to purposes of the motion, of the truth of all material facts
respondent judge dated October 7, 1977, dismissing Civil pay the docket fee in order to perfect the appeal. This was to pleaded in the complaint instituted by the Republic.
Case No. 4062 is set aside, and respondent judge is ordered be followed by another motion for extension filed by the
to require private respondents to file their answer to the Solicitor General, resulting in the Court of Appeals granting the An important factual issue raised in the complaint was the
complaint in said Civil Case No. 4062 and thereafter to petitioner another extension of fifteen days from December 10, classification of the lands as forest lands. This material
proceed with the trial of the case on the merits and to render 1977. Finally before this period of extension lapsed, instead of allegation stated in the Republic's complaint' was never denied
judgment thereon.' an appeal, a petition for certiorari with the respondent Court of specifically 9 by the defendants (petitioners herein) SUNBEAM
Appeals was filed. and CORAL BEACH.
The following facts stated by the respondent Court in its
decision and restated by the petitioners in their petition are According to the Solicitor General, the Court of First Instance If it is true that the lands are forest lands, then all these
accurate: committed grave abuse of discretion in dismissing the proceedings become moot and academic. Land remains
complaint and in unclassified land until it is released therefrom and rendered
(a) On April 29, 1963, the Director of Lands caused the open to disposition.10
issuance of a Sales Patent in favor of defendant Sunbeam a. Not finding that since the lower court acted in a Motion to
Convenience Foods, Inc., over the parcels of land both Dismiss, the correctness of its decision must be decided in the Our adherence to the Regalian doctrine subjects all
situated in Mariveles, Bataan and more particularly described assumed truth and accuracy of the allegations of the agricultural, timber, and mineral lands to the dominion of the
and bounded as follows: complaint. The complaint alleges that the lands in question are State.11 Thus, before any land may be declassified from the
forest lands; hence, inalienable. forest group and converted into alienable or disposable land
Lot 1-Sgs-2409 (area 3,113,695 sq. m ) for agricultural or other purposes, there must be a positive act
b. Finding that Lots I and 2 are alienable and disposable lands from the government. Even rules on the confirmation of
Lot 2-Sgs-2409 area 1,401,855 sq. m of the public domain under the jurisdiction of the Director of imperfect titles do not apply unless and until the land classified
Lands despite clear and positive evidence to the contrary. as forest land is released in an official proclamation to that
(b) On May 3, 1963, the aforesaid Sales Patent was registered effect so that it may form part of the disposable agricultural
with the defendant Register of Deeds of Bataan who in turn c. Concluding that the complaint for reversion is defective as it lands of the public domain. 12
issued Original Certificate of Title No. Sp-24 in favor of was not initiated by the Director of Lands.
defendant Sunbeam Convenience Foods, Inc., for the two The mere fact that a title was issued by the Director of Lands
parcels of land above-described; d. Finding that the complaint for reversion states no cause of does not confer any validity on such title if the property
action for alleged failure of petitioner to exhaust administrative covered by the title or patent is part of the public forest. 13
(c) Subsequently, Original Certificate of Title No. Sp-24 was remedies. 6
cancelled and in lieu thereof, Transfer Certificate of Title No. T- The only way to resolve this question of fact as to the
12421 was issued over Lot 1, Sgs-2409, while Transfer The Court of Appeals gave due course to the petition for classification of the land is by remanding the case to the lower
Certificate of Title No. 12422 was issued over Lot 2, Sgs-2409, certiorari, set aside the Order of Dismissal rendered by the court for a full- dress trial on the issues involved.
both in favor of defendant Coral Beach Development Court of First Instance in Civil Case No. 4062, and ordered the
Corporation I presiding judge Hon. Pedro T. Santiago to receive the answers Generally, the rules of procedure must be observed so that the
of the private respondents SUNBEAM and CORAL BEACH in efficient administration of justice is ensured. However, the
(d) On May 11, 1976, the Solicitor General in the name of the the action for reversion. rules of procedure should be viewed as mere tools designed to
Republic of the Philippines instituted before the Court of First facilitate the attainment of justice.14 They must lead to the
Instance of Bataan, an action for reversion docketed as Civil Hence Sunbeam and Coral Beach filed this petition for review. proper and just determination of litigation, without tying the
Case No. 4062. 2 hands of the law or making it indifferent to realities.1âwphi1
A review is not a matter of right but of sound judicial discretion,
SUNBEAM and CORAL BEACH filed a Motion to Dismiss on and is granted only when there are special and important Certiorari is one such remedy. Considered extraordinary, it is
the following grounds: reasons therefore. The following, while neither controlling nor made available only when there is no appeal, nor any plain,
speedy or adequate remedy in the ordinary course of the law.
2

15 The long line of decisions denying the petition for certiorari, namely: the Heirs of Casiano Sandoval (who had since died), 1) the possessory information title of the applicants and
either before appeal was availed of or specially in instances the Bureau of Lands, the Bureau of Forest Development, the their predecessors-in-interest;
where the appeal period has lapsed, far outnumbers the Heirs of Liberato Bayaua, and the Philippine Cacao and Farm
instances when certiorari was given due course. The few Products, Inc. Under the compromise agreement, the Heirs of 2) the fact that Lot 7454 was never claimed to be public
significant exceptions were: when public welfare and the Casiano Sandoval (as applicants) renounced their claims and land by the Director of Lands in the proper cadastral
advancement of public policy dictate; or when the broader ceded — proceedings;
interests of justice so require, or when the writs issued are null,
16 or when the questioned order amounts to an oppressive 1) in favor of the Bureau of Lands, an area of 4,109 3) the pre-war certification of the National Library dated
exercise of judicial authority. 17 hectares; August 16, 1932 to the effect that the (Estadistica de
Propiedades) of Isabela issued in 1896 and appearing in the
We find nothing disagreeable with the action of the Court of 2) in favor of the Bureau of Forest Development, Bureau of Archives, the property in question was registered
Appeals to give due course to the petition considering that the 12,341 hectares; under the 'Spanish system of land registration as private
issue affected a matter of public concern which is the property owned by Don Liberato Bayaua, applicants'
disposition of the lands of our matrimony No less than the 3) in favor of the Heirs of Liberato Bayaua, 4,000 predecessors-in-interest;
Constitution protects its policy. hectares; and
4) the proceeding for registration, brought under Act
We therefore find no compelling reason to disturb the findings 4) in favor of Philippine Cacao & Farm Products, Inc., 496 (the Torrens Act) presupposes that there is already a title
of the appellate court, in the absence of a clear showing that 8,000 hectares. to be confirmed by the court, distinguishing it from proceedings
the Court of Appeals has decided a question of substance in a under the Public Land Act where the presumption is always
manner inconsistent with jurisprudence, or that the respondent The remaining area of 5,500 hectares was, under the that the land involved belongs to the State.
Court has departed from the accepted and usual course of compromise agreement, adjudicated to and acknowledged as
judicial proceedings. In sum, no reversible error has been owned by the Heirs of Casiano Sandoval, but out of this area, Under the Regalian Doctrine 2 all lands not otherwise
committed by the respondent court. 18 1,500 hectares were assigned by the Casiano Heirs to their appearing to be clearly within private ownership are presumed
counsel, Jose C. Reyes, in payment of his attorney's fees. In to belong to the State. Hence it is that all applicants in land
WHEREFORE, the petition is DENIED and the decision of the consideration of the areas respectively allocated to them, all registration proceedings have the burden of overcoming the
Court of Appeals is affirmed. Costs against the petitioners. the parties also mutually waived and renounced all their prior presumption that the land thus sought to be registered forms
claims to and over Lot No. 7454 of the Santiago Cadastre. part of the public domain. 3 Unless the applicant succeeds in
SO ORDERED. showing by clear and convincing evidence that the property
In a decision rendered on March 5, 1981, the respondent involved was acquired by him or his ancestors either by
G.R. No. L-60413 October 31, 1990 Judge approved the compromise agreement and confirmed the composition title from the Spanish Government or by
REPUBLIC OF THE PHILIPPINES, petitioner, title and ownership of the parties in accordance with its terms. possessory information title, or any other means for the proper
vs. HON. SOFRONIO G. SAYO, Judge, Br. I, C I, Nueva acquisition of public lands, the property must be held to be part
Vizcaya, HEIRS OF CASIANO SANDOVAL, HEIRS OF The Solicitor General, in behalf of the Republic of the of the public domain . 4 The applicant must present competent
LIBERATO BAYAUA, JOSE C. REYES, and PHILIPPINE Philippines, has taken the present recourse in a bid to have and persuasive proof to substantiate his claim; he may not rely
CACAO AND FARM PRODUCTS, INC., respondents. that decision of March 5, 1981 annulled as being patently void on general statements, or mere conclusions of law other than
NARVASA, J.: and rendered in excess of jurisdiction or with grave abuse of factual evidence of possession and title. 5
discretion. The Solicitor General contends that —
Sought to be annulled and set aside in this special civil action In the proceeding at bar, it appears that the principal document
of certiorari is the decision of respondent Judge Sofronio G. 1) no evidence whatever was adduced by the parties in relied upon and presented by the applicants for registration, to
Sayo rendered on March 5, 1981 in Land Registration Case support of their petitions for registration; prove the private character of the large tract of land subject of
No. N-109, LRC Record No. 20850, confirming, by virtue of a their application, was a photocopy of a certification of the
compromise agreement, the title of the private respondents 2) neither the Director of Lands nor the Director of National Library dated August 16, 1932 (already above
over a tract of land. Forest Development had legal authority to enter into the mentioned) to the effect that according to the Government's
compromise agreement; (Estadistica de Propiedades) of Isabela issued in 1896, the
The spouses, Casiano Sandoval and Luz Marquez, filed an property in question was registered under the Spanish system
original application for registration of a tract of land identified 3) as counsel of the Republic, he should have been but of land registration as private property of Don Liberato Bayaua.
as Lot No. 7454 of the Cadastral Survey of Santiago, BL Cad. was not given notice of the compromise agreement or But, as this Court has already had occasion to rule, that
211 (July 17, 1961) and having an area of 33,950 hectares. otherwise accorded an opportunity to take part therein; Spanish document, the (Estadistica de Propiedades,) cannot
The land was formerly part of the Municipality of Santiago, be considered a title to property, it not being one of the grants
Province of Isabela, but had been transferred to Nueva 4) that he was not even served with notice of the made during the Spanish regime, and obviously not
Vizcaya in virtue of Republic Act No. 236. decision approving the compromise; it was the Sangguniang constituting primary evidence of ownership. 6 It is an
Panlalawigan of Quirino Province that drew his attention to the inefficacious document on which to base any finding of the
Oppositions were filed by the Government, through the "patently erroneous decision" and requested him to take private character of the land in question.
Director of Lands and the Director of Forestry, and some immediate remedial measures to bring about its annulment.
others, including the Heirs of Liberato Bayaua.1 In due course, And, of course, to argue that the initiation of an application for
an order of general default was thereafter entered on The respondents maintain, on the other hand, that the Solicitor registration of land under the Torrens Act is proof that the land
December 11, 1961 against the whole world except the General's arguments are premised on the proposition that Lot is of private ownership, not pertaining to the public domain, is
oppositors. 7454 is public land, but it is not. According to them, as pointed to beg the question. It is precisely the character of the land as
out in the application for registration, the private character of private which the applicant has the obligation of establishing.
The case dragged on for about twenty (20) years until March 3, the land is demonstrated by the following circumstances, to For there can be no doubt of the intendment of the Land
1981 when a compromise agreement was entered into by and wit: Registration Act, Act 496, that every applicant show a proper
among all the parties, assisted by their respective counsel, title for registration; indeed, even in the absence of any
3

adverse claim, the applicant is not assured of a favorable [G.R. No. 129401. February 2, 2001] together with other lots acquired by LSBDA with an aggregate
decree by the Land Registration Court, if he fails to establish a FELIPE SEVILLE in his capacity as judicial administrator area of 442, 7508 square meters.
proper title for official recognition. of the estate of JOAQUIN ORTEGA and/or FELIPE
SEVILLE, EMILIA ESTRADA, MARIA S. TELLDER, MA. 4. After due notice and investigation conducted by the Bureau
It thus appears that the decision of the Registration Court a ISABEL SEVILLE, MA. TERESITA LICARDO, FRANCISCO of Lands, Miscellaneous Sales Patent No. 9353 was issued in
quo is based solely on the compromise agreement of the SEVILLE, RAMON O. SEVILLE, JOSE MARIE SEVILLE, the name of [Respondent] LSBDA on the basis of which
parties. But that compromise agreement included private GEMMA ALVAREZ-ASAYAS, ANNABELLE ALVAREZ- Original Certificate of Title No. P-28131 was transcribed in the
persons who had not adduced any competent evidence of their GONZALES, SYLVIA ALVAREZ-LIOK, ADOLFO O. Registration Book for the [P]rovince of Leyte on August 12,
ownership over the land subject of the registration proceeding. ALVAREZ JR., DIANA ALVAREZ-DABON, MARIA 1983 in the name of [Respondent] LSBDA. On December 14,
Portions of the land in controversy were assigned to persons SALVADOR O. POLANCOS and JOAQUIN ORTEGA II as 1989, LSBDA assigned all its rights over the subject property
or entities who had presented nothing whatever to prove their successors-in-interest of JOAQUIN ORTEGA and his to its [Co-respondent] National Development Company (NDC)
ownership of any part of the land. What was done was to estate, petitioners, vs. NATIONAL DEVELOPMENT as a result of which a new Transfer Certificate of Title was
consider the compromise agreement as proof of title of the COMPANY, LEYTE SAB-A BASIN DEVELOPMENT issued on March 2, 1990 by the Registry of Deeds for the
parties taking part therein, a totally unacceptable proposition. AUTHORITY, PHILIPPINE ASSOCIATED SMELTING AND province of Northern Leyte in the name of NDC. The subject
The result has been the adjudication of lands of no little REFINING CORPORATION, LEPANTO CONSOLIDATED property was leased to [Respondents] Philippine Associated
extension to persons who had not submitted any substantiation MINING CO., PHILIPPINE PHOSPHATE FERTILIZER Smelting & Refining Corporation (PASAR), Philippine
at all of their pretensions to ownership, founded on nothing but CORPORATION, CALIXTRA YAP and REGISTER OF Phosphate Fertilizer Corporation (PHILPHOS) and Lepanto
the agreement among themselves that they had rights and DEEDS OF LEYTE, respondents. Consolidated Mining Co., Inc. (LEPANTO).
interests over the land. PANGANIBAN, J.:
5. On November 29, 1988, the Estate of Joaquin Ortega
The assent of the Directors of Lands and Forest Development Unless a public land is shown to have been reclassified as represented by judicial administrator Felipe Seville filed with
to the compromise agreement did not and could not supply the alienable or actually alienated by the State to a private person, the Regional Trial Court (Branch 12) of Ormoc City, a
absence of evidence of title required of the private that piece of land remains part of the public domain. Hence, complaint for recovery of real property, rentals and damages
respondents. occupation thereof, however long, cannot ripen into ownership. against the above-named [respondents] which complaint was
later on amended on May 11, 1990. [Respondents] filed their
As to the informacion posesoria invoked by the private The Case respective Answers. After trial, the trial court rendered
respondents, it should be pointed out that under the Spanish judgment the dispositive portion of which reads as follows:
Mortgage Law, it was considered a mode of acquiring title to Before us is a Petition for Review on Certiorari assailing the
public lands, subject to two (2) conditions: first, the inscription November 29, 1996 Decision of the Court of Appeals[1] (CA), WHEREFORE, [a] decision is hereby rendered for [petitioners]
thereof in the Registry of Property, and second, actual, public, as well as the May 19, 1997 CA Resolution[2] denying the and against [respondents].
adverse, and uninterrupted possession of the land for twenty Motion for Reconsideration. The dispositive part of the CA
(20) years (later reduced to ten [10] years); but where, as here, Decision reads as follows: 1. The Deed of Sale executed by Calixtra Yap on June 14,
proof of fulfillment of these conditions is absent, the 1980 in favor of LSBDA, (Exhibit PP and 25) conveying the
informacion posesoria cannot be considered as anything more WHEREFORE, the appealed decision is REVERSED and SET subject property to said LSBDA is declared NULL and VOID ab
than prima facie evidence of possession. 7 ASIDE. Another judgment is hereby rendered dismissing the initio;
complaint. The counterclaims of appellants are denied. Costs
Finally, it was error to disregard the Solicitor General in the against plaintiffs-appellees.[3] 2. The intestate estate of JOAQUIN ORTEGA is declared the
execution of the compromise agreement and its submission to owner in fee simple of the 735,333 square meters real property
the Court for approval. It is, after all, the Solicitor General, who The Facts subject of the present action and defendant NDC is ordered to
is the principal counsel of the Government; this is the reason segregate the same area from OCT P-28131 and CONVEY
for our holding that "Court orders and decisions sent to the The appellate court narrated the undisputed facts in this the same to the Estate of Joaquin Ortega;
fiscal, acting as agent of the Solicitor General in land manner:
registration cases, are not binding until they are actually 3. Upon the segregation of the 735,333 square meters from
received by the Solicitor General." 8 1. By virtue of Presidential Decree No. 625, Leyte Sab-A Basin OCT No. P-28131 the Register of Deeds of the Province of
Development Authority (LSBDA) was created to integrate Leyte is ordered to issue a new title to the said portion in the
It thus appears that the compromise agreement and the government and private sector efforts for a planned name of the Intestate Estate of Joaquin Ortega;
judgment approving it must be, as they are hereby, declared development and balanced growth of the Sab-a Basin in the
null and void, and set aside. Considerations of fairness [P]rovince of Leyte, empowered to acquire real property in the 4. [Respondents] LSBDA, NDC, PASAR, are ordered to pay
however indicate the remand of the case to the Registration successful prosecution of its business. Letter of Instruction No. jointly and severally to [petitioners] the sum of FOUR MILLION
Court so that the private parties may be afforded an 962 authorized LSBDA to acquire privately-owned lands SEVEN HUNDRED EIGHTY FOUR THOUSAND EIGHT
opportunity to establish by competent evidence their respective circumscribed in the Leyte Industrial Development Estate HUNDRED FORTY SIX PESOS (P4,784,846.00) as rentals
claims to the property. (LIDE) by way of negotiated sales with the landowners. due from 1979 to the present, plus accrued interest pursuant to
par. 2 of the Lease Contract between NDC and PASAR.
WHEREFORE, the decision of the respondent Judge 2. On June 14, 1980, [Respondent] Calixtra Yap sold to (Exhibit 54)
complained of is ANNULLED and SET ASIDE. Land LSBDA Lot No. 057 SWO 08-000047 consisting of 464,920
Registration Case No. N-109 subject of the petition is square meters, located at Barangay Sto. Rosario, Isabel, 5. [Respondents] LSBDA, NDC, and PHILPHOS are also
REMANDED to the court of origin which shall conduct further Leyte, covered under Tax Declarations Nos. 3181, 3579, 3425, ordered to pay jointly and severally [petitioners] the sum of
appropriate proceedings therein, receiving the evidence of the 1292 and 4251 under the name of said vendor. TWO MILLION EIGHTY SIX THOUSAND THREE HUNDRED
parties and thereafter rendering judgment as such evidence NINETY EIGHT PESOS AND SIXTY CENTAVOS
and the law may warrant. No pronouncement as to costs. 3. On June 1, 1982, appellant LSBDA filed a Miscellaneous (P2,086,398.60) as accrued rentals of PHILPHOS from 1979
Sales Application with the Bureau of Lands covering said lot to present, plus the accrued interest for non-payment pursuant
SO ORDERED. to paragraph 2 of the same Lease Contract cited above;
4

6. [Respondents] are ordered to pay jointly and severally The Courts Ruling xxx xxx xxx
[petitioners] P200,000.00 as indemnity for the value of the
ancestral home; The Petition has no merit. (b) those who by themselves or through their predecessor in-
interest have been in open, continuous, exclusive and
7. [Respondents] are also ordered to pay jointly and severally Main Issue: notorious possession and occupation of agricultural lands of
[petitioners] the sum of P250,000.00 as reimbursement for Validity of LSBDAs Title the public domain, under a bona fide claim of acquisition or
attorneys fees and the further sum of P50,000.00 as expenses ownership, for at least thirty years immediately preceding the
for litigation; Petitioners argue that LSBDAs title to 73 hectares of the 402- filing of the application for confirmation of title except when
hectare Leyte Industrial Development Estate was void, having prevented by war or force majeure. They shall be conclusively
8. Finally, [petitioners] and [respondents] are ordered to sit allegedly been obtained from Calixtra Yap who had no right to presumed to have performed all the conditions essential to a
down together and discuss the possibility of a compromise it. They maintain that they acquired title to the disputed Government grant and shall he entitled to a certificate of title
agreement on how the improvements introduced on the property by acquisitive prescription, because they and their under the provisions of this Chapter.
landholding subject of the present suit should be disposed of predecessors in interest had been in possession of it for more
and for the parties to submit to this Court a joint manifestation than thirty years.[7] Although it was the subject of settlement Under Section 4 of Presidential Decree (PD) No. 1073,[10]
relative thereto. In the absence of any such compromise proceedings, petitioners further claim that Yap sold the same paragraph b of the aforecited provision applies only to
agreement, such improvements shall be disposed of pursuant to LSBDA without the permission of the trial court. alienable and disposable lands of the public domain. The
to Article 449 of the New Civil Code. provision reads:
Disputing these contentions, respondents and the appellate
Costs against [respondents]. court maintain that petitioners have not shown that the land SEC. 4. The provisions of Section 48 (b) and Section 48 (c),
had previously been classified as alienable and disposable. Chapter VIII, of the Public Land Act, are hereby amended in
SO ORDERED.[4] Absent such classification, they argue that possession of it, no the sense that these provisions shall apply only to alienable
matter how long, could not ripen into ownership. and disposable lands of the public domain which have been in
Ruling of the Court of Appeals open, continuous, exclusive and notorious possession and
We agree with respondents and the appellate court. First. occupation by the applicant himself or thru his predecessor-in-
Citing the Regalian doctrine that lands not appearing to be There was no showing that the land had been classified as interest, under a bona fide claim of acquisition of ownership,
privately owned are presumed to be part of the public domain, alienable before the title was issued to LSBDA; hence, since June 12, 1945.
the CA held that, first, there was no competent evidence to petitioners could not have become owners thereof through
prove that the property in question was private in character. acquisitive prescription. Second, petitioners challenge to It should be stressed that petitioners had no certificate of title
Second, possession thereof, no matter how long, would not LSBDAs title cannot be granted, because it is based on a over the disputed property. Although they claim that their title
ripen into ownership, absent any showing that the land had wrong premise and amounts to a collateral attack, which is not was based on acquisitive prescription, they fail to present
been classified as alienable. Third, the property had been allowed by law. incontrovertible proof that the land had previously been
untitled before the issuance of the Miscellaneous Sales Patent classified as alienable. They simply brush aside the conclusion
in favor of the LSBDA. Fourth, petitioners were guilty of laches, Public Character of the Land of the CA on this crucial point by saying that it was without
because they had failed to apply for the judicial confirmation of factual basis.[11] Instead, they maintain that the private
their title, if they had any. Fifth, there was no evidence of bad Under the Regalian doctrine, all the lands of the public domain character of the land was evidenced by various tax
faith on the part of LSBDA in dealing with Yap regarding the belong to the State, which is the source of any asserted right to declarations, Deeds of Sale, and Decisions of the trial court
property. ownership of land. All lands not otherwise appearing to be and even the Supreme Court.[12]
clearly within private ownership are presumed to belong to the
Hence, this Petition.[5] State.[8] In Menguito v. Republic,[9] the court held that [u]nless Petitioners arguments are not convincing. Tax declarations are
public land is shown to have been reclassified or alienated to a not conclusive proofs of ownership, let alone of the private
The Issues private person by the State, it remains part of the inalienable character of the land. At best, they are merely indicia of a claim
public domain. Indeed, occupation thereof in the concept of of ownership.[13] In Spouses Palomo v. CA,[14] the Court also
In their Memorandum, petitioners submit the following issues owner, no matter how long, cannot ripen into ownership and be rejected tax declarations as proof of private ownership, absent
for the consideration of the Court:[6] registered as a title. To overcome such presumption, any showing that the forest land in question had been
incontrovertible evidence must be shown by the applicant. reclassified as alienable.
A. Whether or not the sale by Calixtra Yap of the Estate of the Absent such evidence, the land sought to be registered
Late Joaquin Ortega in favor of LSBDA was null and void. remains inalienable. Moreover, the Deeds of Sale of portions of the disputed
property, which Joaquin Ortega and several vendors executed,
B. Whether or not the issuance of a Miscellaneous Sales A person in open, continuous, exclusive an notorious do not prove that the land was private in character. The
Patent and an Original Certificate of Title in favor of LSBDA possession of a public land for more than thirty years acquires question remains: What was the character of the land when
was valid. an imperfect title thereto. That title may be the subject of Ortega purchased it? Indeed, a vendee acquires only those
judicial confirmation, pursuant to Section 48 of the Public Land rights belonging to the vendor. But petitioners failed to show
C. Whether or not petitioners are guilty of laches. Act, which provides: that, at the time, the vendors were already its owners, or that
the land was already classified as alienable.
D. Whether or not petitioners are entitled to the remedy of SECTION 48. The following described citizens of the
reconveyance and the damages awarded by the trial court. Philippines, occupying lands of public domain or claiming to Also misplaced is petitioners reliance on Ortega v. CA,[15] in
own any such lands or an interest thereon, but whose titles which the Supreme Court allegedly recognized the private
In the main, the Court is called upon to determine the validity have not been perfected or completed, may apply to the Court character of the disputed property. In that case, the sole issue
of LSBDAs title. In resolving this issue, it will also ascertain of First Instance of the province where the land is located for was whether the respondent judge xxx acted in excess of
whether, before the issuance of the title, the land was private confirmation of their claims, and the issuance of a certificate of jurisdiction when he converted Civil Case No. 1184-O, an
or public. title therefore, under the Land Registration Act, to wit: action for quieting of title, declaration of nullity of sale, and
5

annulment of tax declaration of a parcel of land, into an action in a public auction, its purchase of the property from the acquired the property by acquisitive prescription; and that Yap,
for the declaration of who is the legal wife, who are the Bureau of Lands. Indeed, Yap merely conveyed a claim, not a without any right or authority, sold the same to LSBDA.
legitimate children, if any, and who are the compulsory heirs of title which she did not have.
the deceased Joaquin Ortega.[16] The Court did not all make Conclusion
any ruling that the property had been classified as alienable. Collateral Attack
In the light of our earlier disquisition, the theory has no leg to
In any event, Ortega arose from a suit for quieting of title, an There is another reason for denying the present Petition. stand on. Absent any showing that the land has been classified
action quasi in rem that was binding only between the Petitioners insist that they are not seeking the re-opening of a as alienable, their possession thereof, no matter how lengthy,
parties.[17] The present respondents as well as the Bureau of decree under the Torrens system. Supposedly, they are only cannot ripen into ownership. In other words, they have not
Lands, which subsequently declared that the land was public, praying for the segregation of 735,333 square meters of land, become owners of the disputed property. Moreover, LSBDAs
are not bound by that ruling, because they were not impleaded or 73 hectares more or less from the OCT No. P-28131 issued title was derived from a Miscellaneous Sales Patent, not from
therein. to LSBDA.[21] This disputation is mere quibbling over the Yap. Finally, petitioners cannot, by a collateral attack,
words, plain and simple. challenge a certificate of title that has already become
While petitioners refer to the trial court proceedings indefeasible and incontrovertible.
supposedly recognizing the private character of the disputed Semantics aside, petitioners are effectively seeking the
property, they make no claim that these cases directly involve modification of LSBDAs OCT, which allegedly encompassed If petitioners believe that they have been defrauded by Yap,
the classification of the land, or that the Bureau of Lands is a even a parcel of land allegedly belonging to them. Hence, the they should seek redress, not in these proceedings, but in a
party thereto. present suit, purportedly filed for the recovery of real property proper action in accordance with law.
and damages, is tantamount to a collateral attack not
Clearly, the burden of proof that the land has been classified sanctioned by law. Section 48 of PD 1529, the Property WHEREFORE, the Petition is hereby DENIED and the
as alienable is on the claimant.[18] In the present case, Registration Decree, expressly provides: assailed Decision AFFIRMED. Costs against petitioners.
petitioners failed to discharge this burden. Hence, their
possession of the disputed property, however long, cannot SEC. 48. Certificate not subject to collateral attack. -- A SO ORDERED.
ripen into ownership. certificate of title shall not be subject to collateral attack. It
cannot be altered, modified, or cancelled except in a direct G.R. No. 73246 March 2, 1993
LSBDAs Title proceeding in accordance with law. DIRECTOR OF LANDS AND DIRECTOR OF FOREST
DEVELOPMENT, petitioners, vs. INTERMEDIATE
Equally unmeritous is the argument of petitioners that the title It has been held that a certificate of title, once registered, APPELLATE COURT AND J. ANTONIO ARANETA,
of LSBDA is void. As earlier stated, they claim that such title should not thereafter be impugned, altered, changed, modified, respondents.
was derived from Calixtra Yap, who was allegedly not the enlarged or diminished, except in a direct proceeding permitted NOCON, J.:
owner of the property. Petitioners assume that LSBDA, having by law. Otherwise, the reliance on registered titles would be
acquired the rights of Yap, resorted to a confirmation of her lost.[22] For review before Us is the decision of the Court of Appeals in
imperfect title under Section 48 of the Public Land Act. This the land registration case entitled J. Antonio Araneta v. The
argument is devoid of factual or legal basis. Moreover, the title became indefeasible and incontrovertible Director of Lands and Director of Forest Development, AC-
after the lapse of one year from the time of its registration and G.R. CV. No. 00636,1 affirming the lower court's approval of
Petitioners fail to consider that the title of LSBDA was based, issuance.[23] Section 32 of PD 1529 provides that [u]pon the the application for registration of a parcel of land in favor of
not on the conveyance made by Yap, but on Miscellaneous expiration of said period of one year, the decree of registration applicant therein, J. Antonio Araneta.
Sales Patent No. 9353 issued by the director of the Bureau of and the certificate of title shall become incontrovertible. Any
Lands. In fact, after LSBDA had filed an application for patent, person aggrieved buy such decree of registration in any case Evidence show that the land involved is actually an island
the Bureau of Lands conducted an investigation and found that may pursue his remedy by action for damages against the known as Tambac Island in Lingayen Gulf. Situated in the
the land was part of the public domain. After compliance with applicant or other persons responsible for the fraud. Although Municipality of Bani, Pangasinan, the area consists of 187,288
the notice and publication requirements, LSBDA acquired the LSBDAs title was registered in 1983, petitioners filed the square meters, more or less. The initial application for
property in a public auction conducted by the Bureau of amended Complaint only in 1990. registration was filed for Pacific Farms, Inc. under the
Lands.[19] provisions of the Land Registration Act, Act No. 496, as
Reconveyance amended.
Petitioners insist, however, that LSBDA was estopped from
claiming that the land was public, because the Deed of Sale Petitioners also claim that the disputed property should be The Republic of the Philippines, thru the Director of Lands
executed by Yap in its favor stipulated that the seller is the reconveyed to them. This cannot be allowed. Considering that opposed the application alleging that the applicant, Pacific
absolute owner in fee simple of the xxx described property.[20] the land was public before the Miscellaneous Sales Patent was Farms, Inc. does not possess a fee simple title to the land nor
It is scarcely necessary to address this point. To begin with, issued to LSBDA, petitioners have no standing to ask for the did its predecessors possess the land for at least thirty (30)
the power to classify a land as alienable belongs to the State, reconveyance of the property to them. The proper remedy is years immediately preceding the filing of application. The
not to private entities. Hence, the pronouncements of Yap or an action for reversion, which may be instituted only by the opposition likewise specifically alleged that the applicant is a
LSBDA cannot effect the reclassification of the property. Office of the Solicitor General, pursuant to section 101 of the private corporation disqualified under the (1973) new
Moreover, the assailed misrepresentation was made by Yap as Public Land Act, which reads as follows: Philippine Constitution from acquiring alienable lands of the
seller. Hence, objections thereto should be raised not by public domain citing Section 11, Article 14.2
petitioners but by LSBDA, the contracting party obviously SEC. 101. All actions for the reversion to the Government of
aggrieved. lands of the public domain or improvements thereon shall be The Director of Forest Development also entered its opposition
instituted by the Solicitor General or the officer acting in his alleging that the land is within the unclassified public land and,
In any case, the actions of LSBDA after Yaps conveyance stead, in the proper courts, in the name of the [Republic] of the hence, inalienable. Other private parties also filed their
demonstrated its position that the disputed land was part of the Philippines. oppositions, but were subsequently withdrawn.
public domain. That this was so can be inferred from LSBDAs Verily, the prayer for reconveyance and, for that matter, the
subsequent application for a Miscellaneous Sales Patent and, entire case of petitioners rest on the theory that they have
6

In an amended application, Pacific Farms, Inc. filed a and registration may be allowed by the court at any stage of
manifestation-motion to change the applicant from Pacific VI. The lower court erred in not declaring that the the proceeding upon just and reasonable terms. 11 On the
Farms, Inc. to J. Antonio Araneta. Despite the supposed applicant has failed to overthrow the presumption that the land other hand, republication is required if the amendment is due
amendment, there was no republication. is a portion of the public domain belonging to the Republic of to substantial change in the boundaries or increase in the area
the Philippines. of the land applied for.
Evidence presented by the applicant include the testimony of
Placido Orlando, fishery guard of Pacific Farms, Inc., who said From the foregoing it appears that the more important issues As to the fourth assignment of error. We do not see any
he has known the disputed land since he attained the age of are: 1) whether the presentation of the tracing cloth plan is relevant dispute in the lower court's application of Presidential
reason for some forty (40) years now; that when he first came necessary; and 2) whether the land known as "Tambac Island" Decree No. 1529, instead of Act No. 496, in adjudicating the
to know the property it was then owned by and in the can be subject to registration. land to the then applicant, assuming that the land involved is
possession of Paulino Castelo, Juan Ambrosio and Julio registrable. Both laws are existing and can stand together.
Castelo, and later on the whole island was bought by Atty. By mere consideration of the first assignment of error, We can P.D. 1529 was enacted to codify the various laws relative to
Vicente Castelo who in turn sold it to J. Antonio Araneta. right away glean the merit of the petition. registration of property, in order to facilitate effective
implementation of said laws.12
Deposition by oral examination of Araneta was also presented, Respondent claims that the tracing cloth plan is with the files of
together with documents of sale, tax declarations and receipts, the Land Registration Commission, and the only evidence that The third, fifth and sixth assignment of errors are likewise
and survey of property. Applicant, however, failed to present can be presented to that fact is the request for the issuance of meritorious and shall be discussed forthwith together.
the tracing cloth plan and instead submitted to the court a certified copy thereof and the certified copy issued pursuant
certified copies thereof. to the request.5 Respondent further argues that failure of the Respondent asserts that contrary to the allegation of
petitioners to object to the presentation of the certified copy of petitioners, the reports of the District Land Officer of Dagupan
While this case is pending here in Court, respondent filed an the tracing cloth plan was the basis of the trial court's denial of City, Land Inspector Perfecto Daroy and Supervising Land
Omnibus Motion for Substitution of private respondent.3 petitioner's motion for reconsideration. Examiner Teodoro P. Nieva show that the subject property is
Apparently, Antonio Araneta had assigned his rights to and an unclassified public land, not forest land. This claim is rather
interest in Tambac Island to Amancio R. Garcia4 who in turn In a very recent decision of this Court, entitled The Director of misleading. The report of Supervising Land Examiner Nieva
assigned his rights and interest in the same property to Johnny Lands v. The Honorable Intermediate Appellate Court and Lino specifically states that the "land is within the unclassified forest
A. Khonghun whose nationality was not alleged in the Anit,6 We have ruled that the submission of the tracing cloth land" under the administrative jurisdiction of the then Bureau of
pleadings. plan is a mandatory requirement for registration. Reiterating Forest Development.13 This was based on the reports of Land
Our ruling in Director of Lands v. Reyes,7 We asserted that Inspector Daroy and District Land Officer Feliciano Liggayu.
On October 4, 1979, the trial court rendered a decision failure to submit in evidence the original tracing cloth plan is
adjudicating the subject property to J. Antonio Araneta. On fatal it being a statutory requirement of mandatory character. Lands of the public domain are classified under three main
appeal to the then Intermediate Appellate Court, the decision categories, namely: Mineral, Forest and Disposable or
of the lower court was affirmed on December 12, 1985. It is of no import that petitioner failed to object to the Alienable Lands.14 Under the Commonwealth Constitution,
presentation of the certified copy of the said plan. What is only agricultural lands were allowed to be alienated. Their
Petitioners raised the following errors: required is the original tracing cloth plan of the land applied for disposition was provided for under Commonwealth Act No. 141
and objection to such requirement cannot be waived either (Secs. 6-7), which states that it is only the President, upon the
I. The lower court erred in adjudicating the lands expressly or impliedly.8 This case is no different from the case recommendation of the proper department head, who has the
subject of registration to applicant-appellee despite his failure of Director of Lands v. Reyes, supra wherein We said that if authority to classify the lands of the public domain into
to present the original tracing cloth plan the submission of the original tracing cloth plan was indeed with the Land alienable or disposable, timber and mineral lands. Mineral and
which is a statutory requirement of mandatory character. Registration Commission, there is no reason why the applicant Timber or forest lands are not subject to private ownership
cannot easily retrieve the same and submit it in evidence, it unless they are first reclassified as agricultural lands and so
II. The lower court erred in not denying registration in being an essential requirement for registration. released for alienation. 15 In the absence of such
favor of J. Antonio Araneta since the amendment of the classification, the land remains as unclassified land until
application was simply an attempt to avoid the application of As to the second assignment of error, We are inclined to agree released therefrom and rendered open to disposition. Courts
the constitutional provision disqualifying a private corporation with petitioners that the amendment of the application from the have no authority to do so. 16
— the Pacific Farms, Inc. in this case — from acquiring lands name of Pacific Farms Inc., as applicant, to the name of J.
of public domain. Antonio Araneta Inc., was a mere attempt to evade This is in consonance with the Regalian doctrine that all lands
disqualification. Our Constitution, whether the 19739 or of the public domain belong to the State, and that the State is
III. The lower court erred in not declaring the land 1987, 10 prohibits private corporations or associations from the source of any asserted right to ownership in land and
known as the "Tambac Island" not subject of registration it holding alienable lands of the public domain except by lease. charged with the conservation of such patrimony. Under the
being an island formed on the seas. Apparently realizing such prohibition, respondent amended its Regalian Doctrine, all lands not otherwise appearing to be
application to conform with the mandates of the law. clearly within private ownership are presumed to belong to the
IV. The lower court erred in adjudicating the land to the State. Hence, a positive act of the government is needed to
applicant under the provisions of Presidential Decree No. However, We cannot go along with petitioners' position that the declassify a forest land into alienable or disposable land for
1529, otherwise known as the Property Registration Decree, absence of republication of an amended application for agricultural or other purposes. 17
despite absence of any specific invocation of this law in the registration is a jurisdictional flaw. We should distinguish. The burden of proof in overcoming the presumption of state
original and amended application. Amendments to the application may be due to change in ownership of the lands of the public domain is on the person
parties or substantial change in the boundaries or increase in applying for registration that the land subject of the application
V. The lower court erred in not granting the the area of the land applied for. is alienable or disposable.18
government's motion for reconsideration at least to enable it to
present proof of the status of the land as within the unclassified In the former case, neither the Land Registration Act, as Unless the applicant succeeds in showing by convincing
public forest, and hence beyond the court's jurisdiction to amended, nor Presidential Decree No. 1529, otherwise known evidence that the property involved was acquired by him or his
adjudicate as private property. as the Property Registration Decree, requires republication ancestors either by composition title from the Spanish
7

Government or by possessory information title, or any other unclassified until released therefrom and rendered open to
means for the proper acquisition of public lands, the property disposition. 24 2. REMEDIAL LAW; CIVIL PROCEDURE; JUDICIAL
must be held to be part of the public domain. The applicant RELIEF; NOT A VESTED RIGHT; A MERE STATUTORY
must present evidence and persuasive proof to substantiate In fairness to respondent, the petitioners should seriously PRIVILEGE, NOT A PROPERTY RIGHT. — There can be no
his claim. 19 consider the matter of the reclassification of the land in vested right in a judicial relief for this is a mere statutory
question. The attempt of people to have disposable lands they privilege and not a property right. The distinction between
In this particular case, respondent presented proof that as have been tilling for generations titled in their name should not statutory privileges and vested rights must be borne in mind for
early as 1921, the subject property has been declared for tax only be viewed with understanding attitude, but as a matter of persons have no vested rights in statutory privileges. The state
purposes with receipts attached, in the names of respondent's policy encouraged. 25 may change or take away rights which were created by the law
predecessors-in-interest. Nevertheless, in that span of time of the state, although it may not take away property which was
there had been no attempt to register the same either under WHEREFORE, the petition is hereby GRANTED and the vested by virtue of such rights.(16A Am. Jur. 2d, pp. 652-653)
Act 496 or under the Spanish Mortgage Law. It is also rather decisions of the courts a quo are REVERSED. Besides, the right to judicial relief is not a right which may
intriguing that Vicente Castelo who acquired almost 90% of the constitute vested right because to be vested, a right must have
property from Alejo Ambrosia, et al. on June 18, 1958 and from SO ORDERED. become a title, legal or equitable, to the present or future
Julio Castelo on June 19, 1958 immediately sold the same to enjoyment of property, or to the present or future enforcement
applicant J. Antonio Araneta on 3 July 1958. G.R. No. 63786-87. April 7, 1993. of a demand or legal exemption from a demand made by
another. (National Carloading Corporation v. Phoenix-E1 Paso
According to the report of Land Investigator Daroy, the land UNITED PARACALE MINING COMPANY, INC., AND COCO Express, Inc., cited in 16A Am, Jur. 2d, p. 651) Definitely, the
was declared for taxation purposes in the name of Vicente GROVE, INC., petitioners, judicial relief herein referred to by the petitioner does not fall
Castelo only in 1958 and the purported old tax declarations are vs. under any of these.
not on file with the Provincial Assessor's Office. HON. JOSELITO DELA ROSA, in his capacity as the former
Judge of the Court of First Instance of Camarines Norte, 3. CIVIL LAW; PRESIDENTIAL DECREE 1214; A
In any case tax declarations and receipts are not conclusive Branch 2, et al., respondents. VALID EXERCISE OF THE SOVEREIGN POWER OF THE
evidence of ownership or of the right to possess land when not STATE AS OWNER OF LAND OF PUBLIC DOMAIN;
supported by evidence.20 The fact that the disputed property G.R. No. 70423. April 7, 1993. SUSTAINED IN CASE AT BAR. — The heart of these twin
may have been declared for taxation purposes in the names of ZAMBALES CHROMITE MINING COMPANY, INC., petitions is the question of constitutionality of P.D. 1214.
the applicants or of their predecessors-in-interest way back in petitioner, vs. HON. ALFREDO L. BENIPAYO, as Judge, Unless P.D. 1214 is successfully assailed, the petitioners will
1921 does not necessarily prove ownership. They are merely Regional Trial Court of Manila, and PHILZEA MINING & be but mere applicants for the lease of the mining claims
indicia of a claim of ownership.21 DEVELOPMENT CORPORATION, respondents. involved and would thus have no causes of action against
private respondents. This question has been resolved by this
Respondent's contention that the BFD, LC Map No. 681, G.R. No. 73931. April 7, 1993. Court in Santa Rosa Mining Company, Inc. vs. Leido, Jr. [156
certified on August 8, 1927 which was the basis of the report JOSEPH V. LOPEZ and MIGUEL C. ANDRADE, petitioners, SCRA 1 (1987), which ruling was reiterated in Zambales
and recommendation of the Land Examiner, is too antiquated; vs. THE INTERMEDIATE APPELLATE COURT MARSMAN Chromite Mining Company, Inc. vs. Leido, Jr., 176 SCRA 602
that it cannot be conclusively relied upon and was not even & COMPANY, INC. and UNITED PARACALE MINING (1989)] thus: "(W)e hold that Presidential Decree No. 1214 is
presented in evidence, is not well taken. As We have said in COMPANY, INC., respondents. not unconstitutional.' It is a valid exercise of the sovereign
the case of Director of Lands v. CA:22 power of the State, as owner, over lands of the public domain,
SYLLABUS of which petitioner's mining claims still form a part, and over
And the fact that BF Map LC No. 673 dated March 1, 1927 the patrimony of the nation, of which mineral deposits are a
showing subject property to be within unclassified region was 1. STATUTORY CONSTRUCTION; valuable asset. It may be underscored, in this connection, that
not presented in evidence will not operate against the State INTERPRETATION OF A STATUTE; PROPER ONLY WHEN the Decree does not cover all mining claims located under the
considering the stipulation between the parties and under the THERE IS DOUBT OR AMBIGUITY IN ITS LANGUAGE; Phil. Bill of 1902, but only those claims over which their
well-settled rule that the State cannot be estopped by the CASE AT BAR. — The view of the petitioner that by virtue of locators had failed to obtain a patent. And even then, such
omission, mistake or error of its officials or agents, if omission the registration of the mining claims under the Philippine Bill of locators may still avail of the renewable twenty-five year (25)
there was, in fact. 1902 and Act No. 624, the mining claims became private lease prescribed by Pres. Decree No. 463, the Mineral
property and thereby brought outside the control and Development Resources Decree of 1974. Mere location does
Respondent even admitted that Tambac Island is still an supervision of the Director of Mines is without legal basis. The not mean absolute ownership over the affected land or the
unclassified public land as of 1927 and remains to be abovecited law does not distinguish between private property mining claim. It merely segregates the located land or area
unclassified. and lands of the public domain. The provision of law involved from the public domain by barring other would-be locators from
is clear and is not susceptible to interpretation. A condition sine locating the same and appropriating for themselves the
Since the subject property is still unclassified, whatever qua non before the court may construe or interpret a statute is minerals found therein. To rule otherwise would imply that
possession that there be doubt or ambiguity in its language. Section 7 of location is all that is needed to acquire and maintain rights over
the applicant may have had and however long, cannot ripen P.D. 1281 quoted above defining the original and exclusive a located mining claim. This, we cannot approve or sanction
into private ownership. 23 The conversion of subject property jurisdiction of the Director of Mines is clear. Time and again, it because it is contrary to the intention of the lawmaker that the
does not automatically render the property as alienable and has been repeatedly declared by this Court that where the law locator should faithfully and consistently comply with the
disposable. speaks in clear and categorical language, there is no room for requirements for annual work and improvements in the located
interpretation. There is only room for application. [Cebu mining claim. This, we cannot approve or sanction because it
In effect what the courts a quo have done is to release the Portland Cement Company vs. Municipality of Naga, Cebu, 35 is contrary to the intention of the lawmaker that the locator
subject property from the unclassified category, which is SCRA 708 (1968)] Where the law is clear and unambiguous, it should faithfully and consistently comply with the requirements
beyond their competence and jurisdiction. We reiterate that the must be taken to mean exactly what it says and the court has for annual work and improvements in the located mining claim.
classification of public lands is an exclusive prerogative of the no choice but to see to it that its mandate is obeyed. Presidential Decree No. 1214 is in accord with Sec. 8, Art. XIV
Executive Department of the Government and not of the [Chartered Bank Employees Association vs. Ople, 138 SCRA of the 1973 Constitution. The same Constitutional mandate is
Courts. In the absence of such classification, the land remains 273 (1985)]. found in Sec. 2, Art. XII of the 1987 Constitution.
8

CAMPOS, JR., J p: 1984, he showed his "bias, prejudice and personal hatred of conditions thereof, with prayer for the issuance of a preliminary
and hostility to the [petitioners'] counsel [Atty. Pedro A. injunction and/or temporary restraining order. The trial court,
The cases herein were consolidated upon the representations Venida]." however, upon motion of the defendant therein, dismissed the
of petitioners that they involve the same issues or questions of case.
law or at the very least, revolve around the same set of facts. A On December 24, 1984 the petitioners filed a motion for a
perusal of the records, however, reveals the contrary. Only two preliminary hearing on their defense that the RTC lacked Petitioner wants Us to construe Section 7 of P.D. 1281 as
petitions are properly consolidated. Thus, it behooves Us to jurisdiction under PD 605 to issue a temporary restraining applicable only to mineral lands forming part of the public
discuss the cases separately. order or injunction in cases involving or growing out of the domain and not to mining claims located and registered under
action of administrative officials on the applications for permits Philippine Bill of 1902 and Act No. 624 as is its case.
In blatant violation of Section 2, Rule 45 of the Rules of Court for the disposition, exploitation, utilization, or exploration or
which in part, provides: development of the natural resources. Accordingly the RTC, in Section 7 of P.D. 1281 reads as follows:
its order of September 5, 1985, suspended the hearing of the
"Sec. 2. Contents of the petition — The petition shall contain case until the resolution of the petitioners' motion to dismiss. It Sec. 7. In addition to its regulatory and adjudicative
a concise statement of the matters involved, the assignment of is at this point that the present petition was filed." 1 functions over companies, partnerships or persons engaged in
errors made in the court below, and the reasons relied on for mining exploration, development and exploitation, the Bureau
the allowance of the petition, . . . (Emphasis Ours.). The respondent Court denied this petition on grounds that: (1) of Mines shall have original and exclusive jurisdiction to hear
the questions being raised are not proper in a petition for and decide cases involving:
"Only questions of law may be raised in the petition and must certiorari under Rule 65 but rather defenses which should be
be distinctly set forth . . ." raised in the action itself; (2) the question of jurisdiction which (a) a mining property subject of different agreements
has yet to be resolved by the trial court pending resolution of entered into by the claim holder thereof with several mining
this petition devotes nine (9) pages under the subtitle the motion to dismiss is prematurely raised; and (3) there was operators;
"Summary statement of the matters involved" to a discussion no basis for determining whether or not the judge must be
of matters off tangent from the real issues in the case. disqualified. 2 (b) complaints from claimowners that the mining
Definitely, the question of whether or not the Court of Appeals property subject of an operating agreement has not been
erred in ruling that the Regional Trial Court did not commit The review of this decision is what is on appeal before. placed into actual operations within the period stipulated
grave abuse of discretion in issuing an order suspending therein; and
hearing pending the resolution of their motion to dismiss, does We refuse to be persuaded by the petitioners that the RTC
not involve the Philippine Bill of 1902, Executive Order No. must be enjoined from exercising its jurisdiction in settling the (c) cancellation and/or reinforcement of mining
141, much less P.D. 1214. The counsel for petitioners even case presented before it for the reason that the constitutionality contracts due to the refusal of the claimowner/operator to
discussed pending cases in this Court (G.R. No. 63786 - 87 of the law involved in the said case is being questioned before aside by the terms and conditions thereof.
and 69203) which have completely nothing to do with the this Court. This case should have been disposed of
instant petition except for the fact that the parties therein are independently of the other petitions herein. All actions and decisions of the Director of Mines on the above
being represented by the same counsel as in this petition. In cases are subject to review, motu proprio or upon appeal by
several pleadings subsequent to their petition, petitioners The respondent Court of Appeals committed no reversible any person aggrieved thereby, by the Secretary of Natural
insisted that the proceedings in the court below must be error. Neither did it commit grave abuse of discretion as what Resources whose decision shall be final and executory after
restrained until this Court resolves the pending cases petitioners want this Court to believe. The petitioners fail to the lapse of thirty (30) days from receipt by the aggrieved party
abovecited. For this reason this case was consolidated thereto. point out any assigned error which the respondent Court had of said decision, unless appealed to the President in
supposedly committed but simply narrate the action taken by it. accordance with the applicable provisions of Presidential
A summary of the real matters involved in this petition is found Much less have they stated the reasons relied upon for the Decree No. 309 and Letter of Instructions Nos. 119 and 135.
in the respondent Court's decision, to wit: allowance of the instant petition. For being insufficient in
substance and in form, the instant petition lacks merit and The view of the petitioner that by virtue of the registration of
"This is a petition for certiorari and prohibition to enjoin the must be dismissed. the mining claims under the Philippine Bill of 1902 and Act No.
Regional Trial Court, Branch XL, in Camarines Norte from 624, the mining claims became private property and thereby
issuing a writ of preliminary injunction in Civil Case No. 5148 G.R. No. 70423 brought outside the control and supervision of the Director of
and to disqualify the respondent judge from acting in that case. Mines is without legal basis. The abovecited law does not
The case was brought by the respondents Marsman and Co., This is a petition involving the question of jurisdiction of regular distinguish between private property and lands of the public
Inc. and United Paracale Mining, Inc., to enjoin the petitioners, courts in cases which had been placed under the original and domain. The provision of law involved is clear and is not
Joseph V. Lopez and Miguel Andrade, from entering and exclusive jurisdiction of the Bureau of Mines under P.D. 1281. susceptible to interpretation. A condition sine qua non before
conducting mining operations within the "McDonald" and "San the court may construe or interpret a statute is that there be
Antonio" Tunnels in Paracale, Camarines Norte, in which the This petition seeks to reverse the order of then Judge, now doubt or ambiguity in its language. 5 Section 7 of P.D. 1281
private respondents have mining claims ("Tulisan," "Santa Associate Justice of the Court of Appeals, Hon. Alfredo L. quoted above defining the original and exclusive jurisdiction of
Marta," "California," and "Rocky Mountain Fraction"). On Benipayo, dismissing the complaint filed by petitioner herein on the Director of Mines is clear. Time and again, it has been
December 11, 1984 the RTC issued a restraining order against the ground of lack of jurisdiction, citing Section 7 of P.D. 1281 repeatedly declared by this Court that where the law speaks in
the petitioners. and the doctrine enunciated in Twin Peaks Mining Association, clear and categorical language, there is no room for
et al. vs. Navarro, 3 that an action for the enforcement of interpretation. There is only room for application. 6 Where the
On December 12 the petitioners filed their answer alleging mining contracts, in this case cancellation of a mining contract, law is clear and unambiguous, it must be taken to mean
that, in accordance with PD 1214, the private respondents had is outside the competence of regular courts in view of the law exactly what it says and the court has no choice but to see to it
forfeited their right to the mining claims. They likewise argued cited. 4 that its mandate is obeyed. 7
that in view of PD 605, the RTC had no jurisdiction to entertain
the case. On the same day the petitioners moved for the The complaint filed with the then CFI of Manila, Branch XVI, This Court in Benguet Corporation vs. Leviste, 8 made these
disqualification of the respondent judge of the RTC, claiming was one for the rescission of its mining contract with herein pronouncements:
(sic) that in issuing the restraining order of December 11, private respondent on grounds of violations of the terms and
9

"We grant the petition. Presidential Decree No. 1281 which The heart of these twin petitions is the question of natural resources shall be under the full control and
took effect on January 16, 1978 vests the Bureau of Mines with constitutionality of P.D. 1214. Unless P.D. 1214 is successfully supervision of the State . . .'"
jurisdictional supervision and control over all holders of mining assailed, the petitioners will be but mere applicants for the
claims or applicants for and/or grantees of mining licenses, lease of the mining claims involved and would thus have no Notwithstanding Our ruling , in favor of the constitutionality of
permits, leases and/or operators thereof, including mining causes of action against private respondents. P.D. 1214, petitioners contend that having filed mining lease
service contracts and service contractors insofar as their applications on the mining claims they have previously located
mining activities are concerned. To effectively discharge its This question has been resolved by this Court in Santa Rosa and registered under then existing laws, pursuant to the
task as the Government's arm in the administration and Mining Company, Inc. vs. Leido, Jr. 11 thus: requirements of this Presidential Decree, and despite the
disposition of mineral resources, Section 7 of P.D. 1281 waiver of their rights to the issuance of mining patents therefor
confers upon the Bureau quasi-judicial powers as follows: "(W)e hold that Presidential Decree No. 1214 is not (emphasis theirs), they cannot be placed in equal footing with
unconstitutional. ** It is a valid exercise of the sovereign power those who forfeit all rights by virtue of non-filing of an
xxx xxx xxx of the State, as owner, over lands of the public domain, of application within the prescribed period such that they
which petitioner's mining claims still form a part, and over the (petitioners) have no causes of action against private
Analyzing the objective of P.D. 1281, particularly said Section patrimony of the nation, of which mineral deposits are a respondents.
7 thereof, the Court in Twin Peaks Mining Association, the valuable asset. It may be underscored, in this connection, that
case relied upon by petitioner, noted that the trend is to make the Decree does not cover all mining claims located under the We are not persuaded by this contention.
the adjudication of mining cases a purely administrative matter. Phil. Bill of 1902, but only those claims over which their
This observation was reiterated in the more recent case of locators had failed to obtain a patent. And even then, such Although We may agree that those who filed their mining lease
Atlas Consolidated Mining & Development Corporation vs. locators may still avail of the renewable twenty-five year (25) applications have better rights than those who forfeited all their
Court of Appeals." lease prescribed by Pres. Decree No. 463, the Mineral right by not filing at all, this, however, does not amount to any
Development Resources Decree of 1974. vested right which could be the basis for their cause of action
The petitioner further argues that to hold that P.D. 1281 against herein private respondents. What is precisely waived is
retroactively applies to its mining claims which according to it is Mere location does not mean absolute ownership over the their right to the issuance of a mining patent upon application.
private property would constitute impairment of vested rights affected land or the mining claim. It merely segregates the This in effect grants the government the power, in the exercise
since by shifting the forum of the petitioner's case from the located land or area from the public domain by barring other of its sound discretion, to award the patent to the applicant
courts to the Bureau of Mines, as urged by private respondent, would-be locators from locating the same and appropriating for most qualified to explore, develop and exploit the mineral
the substantive rights to full protection of its property rights themselves the minerals found therein. To rule otherwise resources of the country in line with the objectives of P.D. 463,
shall be greatly impaired and prejudiced. The judicial relief would imply that location is all that is needed to acquire and and not necessarily to the original locator of the mining claim.
available for the redress of private property rights violated, now maintain rights over a located mining claim. This, we cannot To sustain their contention that they can question the award of
being enjoyed by petitioner shall be lost altogether. approve or sanction because it is contrary to the intention of mining patents to applicants other than them would put to
the lawmaker that the locator should faithfully and consistently naught the objectives of P.D. 1214 as enunciated in its
This argument does not merit Our approval. There can be no comply with the requirements for annual work and WHEREAS clauses.
vested right in a judicial relief for this is a mere statutory improvements in the located mining claim.
privilege and not a property right. The distinction between We agree with the trial court that with the waiver of their right
statutory privileges and vested rights must be borne in mind for Presidential Decree No. 1214 is in accord with Sec. 8, Art. XIV to the issuance of a mining patent upon their application for a
persons have no vested rights in statutory privileges. The state of the 1973 Constitution which states: mining lease, their status is reduced to a mere applicant, their
may change or take away rights which were created by the law only advantage over the others is the fact that they have
of the state, although it may not take away property which was 'All lands of the public domain, waters, minerals, coal, already conducted explorations at the site and this exploration
vested by virtue of such rights. 9 Besides, the right to judicial petroleum, and other mineral oils, all forces of potential energy, may he ongoing. But still, this credential, so to speak, is not
relief is not a right which may constitute vested right because fisheries, wildlife, and other natural resources of the intended to tie the hands of the government so as to prevent it
to be vested, a right must have become a title, legal or Philippines belong to the State. With the exception of from awarding the mining patent to some other applicants,
equitable, to the present or future enjoyment of property, or to agricultural, industrial or commercial, residential and which in its belief may he more qualified than them.
the present or future enforcement of a demand or legal resettlement lands of the public domain, natural resources
exemption from a demand made by another. 10 Definitely, the shall not be alienated, and no license, concession, or lease for WHEREFORE, the petition in G.R. No. 73931 is hereby
judicial relief herein referred to by the petitioner does not fall the exploration, development, and exploitation, or utilization of DISMISSED for lack of merit; the Order of dismissal assailed in
under any of these. any of the natural resources shall be granted for a period G.R. No. 70423 is AFFIRMED and this petition is hereby
exceeding twenty-five years, renewable for not more than likewise DISMISSED; the Order of dismissal assailed in G.R.
The case at bar falls within the original and exclusive twenty-five years, except as to water rights for irrigation, water Nos. 63786-87 is AFFIRMED and these petitions are hereby
jurisdiction of the Bureau of Mines, hence, the trial court did supply, fisheries, or industrial uses other than development of DISMISSED. No pronouncements as to costs.
not err in dismissing the petitioner's complaint on the ground of water power, in which cases, beneficial use may be the
lack of jurisdiction. measure and the limit of the grant.' SO ORDERED.

G.R. Nos. 63786-87 The same Constitutional mandate is found in Sec. 2, Art. XII of G.R. No. 73974 May 31, 1995
the 1987 Constitution, which declares: REPUBLIC OF THE PHILIPPINES (Represented by the
In these petitions filed by petitioners United Paracale Mining Director of Lands), petitioner,
Company, Inc. and Coco Grove, Inc., petitioners seek to set 'All lands of the public domain, waters, minerals, coal, vs. THE REGISTER OF DEEDS OF QUEZON, MANUEL G.
aside the Order of dismissal of the case they filed with the trial petroleum, and other mineral oils, all forces of potential energy, ATIENZA, DEVELOPMENT BANK OF THE PHILIPPINES
court for the ejectment of their respective defendants from the fisheries, forests or timber, wildlife, flora and fauna, and other (Lucena Branch) and INTERMEDIATE APPELLATE
mining claims which were allegedly privately owned by them natural resources are owned by the State. With the exception COURT, respondents.
having been located and perfected under the provisions of the of agricultural lands, all other natural resources shall not be ROMERO, J.:
Philippine Bill of 1902 and Act No. 624. alienated. The exploration, development, and utilization of
10

This petition for review on certiorari seeks to nullify and set On July 27, 1981, the lower court rendered a decision with the that the land in question is part of the alienable and disposable
aside the decision 1 of the then Intermediate Appellate Court categorical finding based on "solid evidence" that "the land in public land in complete disregard of the trial court's finding that
reversing the decision of the former Court of First Instance of question was found definitely within the forest zone it forms part of the unclassified public forest zone; and (c) the
Quezon, Branch II at Lucena City 2 which annulled Original denominated as Project 21-A." appellate court erred in declaring that the land in question
Certificate of Title (OCT) No. P-13840 and Free Patent (FP) could be alienated and disposed of in favor of Atienza.
No. 324198 issued to Manuel Atienza for a 17-hectare piece of The dispositive portion thereof reads as follows:
land which turned out to be within the forest zone in Pagbilao, We find for the petitioner.
Quezon. WHEREFORE, in view of the foregoing, (J)udgment is hereby
rendered: Appeal is an essential part of our judicial system. As such,
On April 18, 1967, Atienza was awarded FP No. 324198 over a courts should proceed with caution so as not to deprive a party
parcel of land located in Ila, Malicboy, Pagbilao, Quezon, with (a) Declaring as null and void Original Certificate of Title of the right to appeal, particularly if the appeal is meritorious. 3
an area of 172,028 square meters. By virtue of such award, he No. P-13840 in the name of defendant Manuel G. Atienza, as Respect for the appellant's right, however, carries with it the
was issued on May 5, 1967, OCT No. P-13840. well as Free Patent No. V-324198; corresponding respect for the appellee's similar rights to fair
play and justice. Thus, appeal being a purely statutory right, an
Sometime in 1968, an investigation was conducted by the (b) Ordering defendant Manuel G. Atienza to pay the appealing party must strictly comply with the requisites laid
Bureau of Lands in connection with alleged land grabbing Development Bank of the Philippines, Lucena City Branch, the down in the Rules of Court. 4
activities in Pagbilao. It appeared that some of the free sum of P15,053.97, and all interests due thereon; and
patents, including that of Atienza's, were fraudulently acquired. Of paramount importance is the duty of an appellant to serve a
Thus, on March 19, 1970, a criminal complaint for falsification (c) Ordering defendant Manuel G. Atienza to pay the copy of his brief upon the appellee with proof of service
of public documents was filed in the then Court of First costs of this suit. thereof. 5 This procedural requirement is consonant with
Instance of Quezon, Branch II, against Atienza and four other Section 2 of Rule 13, which mandates that all pleadings and
persons for allegedly falsifying their applications for free SO ORDERED. papers "shall be filed with the court, and served upon the
patent, the survey plans, and other documents pertinent to parties affected thereby." The importance of serving copies of
said applications. On appeal, Atienza maintained that the land in question was the brief upon the adverse party is underscored in Mozar v.
not within the unclassified public forest land and therefore Court of Appeals, 6 where the Court held that the appellees
In its decision dated October 4, 1972, the court acquitted the alienable land of the public domain. The then Intermediate "should have been given an opportunity to file their appellee's
accused of the crime charged but, finding that the land covered Appellate Court relied only on the arguments he raised since brief in the Court of Appeals if only to emphasize the necessity
by the application for free patent of private respondent was petitioner had not filed any brief, and arrived at the conclusion of due process."
within the forest zone, declared as null and void OCT No. P- that "(t)he litigated land is part of public land alienable and
13840 in Atienza's name and ordered the Register of Deeds of disposable for homestead and [F]ree Patent." On December In this case, however, the Court of Appeals, oblivious of the
Quezon to cancel the same. 27, 1985, the appellate court set aside the lower court's fact that this case involves public lands requiring as it does the
decision, declared as valid and subsisting Atienza's OCT, and exercise of extraordinary caution lest said lands be dissipated
Meanwhile, before the promulgation of said decision, or on dismissed the cross-claim of the DBP. and erroneously alienated to undeserving or unqualified private
May 10, 1972, then Acting Solicitor General Conrado T. individuals, decided the appeal without hearing the
Limcaoco filed for the petitioner a complaint against Atienza, After receiving a copy of said decision, Assistant Solicitor government's side.
the Register of Deeds of Quezon, and the Rural Bank of General Oswaldo D. Agcaoili informed the Director of Lands of
Sariaya, which was later dropped as defendant and, in an the adverse decision of the appellate court, which noted that Atienza avers that he furnished Atty. Francisco Torres, a
amended complaint, substituted by the Development Bank of no appellee's brief had been filed in said court. Agcaoili also lawyer in the Bureau of Lands and designated special attorney
the Philippines as actual mortgagee of the subject parcel of stated that the Office of the Solicitor General (OSG) had not for the Office of the Solicitor General, with two copies of the
land. Docketed as Civil Case No. 7555, the complaint prayed been furnished with the appellant's brief; that the Bureau of appellant's brief, thereby implying that it was not his fault that
for the declaration of nullity of FP No. 324198 and OCT No. P- Lands received notice of hearing of the record on appeal filed petitioner failed to file its appellee's brief.
13840. by the appellant but the OSG had not been informed of the
"action taken thereon;" that since the Bureau of Lands had Such an assertion betrays a lack of comprehension of the role
In his answer, Atienza claimed that the land in question was no been furnished directly with relevant pleadings and orders, the of the Solicitor General as government counsel or of the OSG
longer within the unclassified public forest land because by the same office should "take immediate appropriate action on the as the government's "law office." 7 Only the Solicitor General,
approval of his application for free patent by the Bureau of decision;" and that it may file a motion for reconsideration as the lawyer of the government, can bring or defend actions
Lands, the land "was already alienable and disposable public within fifteen (15) days from January 6, 1986, the date of on behalf of the Republic of the Philippines and, therefore,
agricultural land." Since the subject land was a very small receipt by the OSG of the copy of the decision sought to be actions filed in the name of the Republic, if not initiated by the
portion of Lot 5139 of the Pagbilao Cadastre, an area which appealed. Solicitor General, will be summarily dismissed. 8 Specifically,
had been declared disposable public land by the cadastral he is empowered to represent the Government in all land
court on March 9, 1932 in Cadastral Case No. 76 entitled "El On January 28, 1986, petitioner filed a motion for extension of registration and related proceedings, 9 such as, an action for
Govierno Filipino de las Islas Filipinas contra Jose Abastillas, time to file motion for reconsideration which was denied in a cancellation of title and for reversion of a homestead to the
et al., G.L.R.O. cadastral Record No. 1124," he also averred resolution dated February 12, 1986. Petitioner's motion for government. 10 Hence, he is entitled to be furnished with
that the Director of Lands had given due course to free and reconsideration of said resolution was likewise denied. copies of all court orders, notices and decisions.
homestead patent applications of claimants to Lot 5139. He Consequently, service of decisions on the Solicitor General is
further alleged that through a certain Sergio Castillo, he had The instant petition for review on certiorari raises the following the proper basis for computing the reglementary period for
been in possession of the land since the Japanese occupation, arguments: (a) petitioner was denied due process and fair play filing appeals and for finality of decisions. His representative,
cultivating it and introducing improvements thereon. The DBP, when Atienza did not furnish it with a copy of his appellant's who may be a lawyer from the Bureau of Lands, has no legal
after due and proper investigation and inspection of his title, brief before the then Intermediate Appellate Court thereby authority to decide whether or not an appeal should be made.
even granted him a loan with the subject property as collateral. depriving it of the opportunity to rebut his assertions which 11
Finally, he stated that his acquittal in the criminal case proved later became the sole basis of the assailed decision of
that he committed no fraud in his application for free patent. December 27, 1985; (b) the appellate court erred in holding
11

Service of the appellant's brief on Atty. Torres was no service private claimant 21 who, in this case, is Atienza. The records "A parcel of land, situated at Baas, Lantawan Basilan.
at all upon the Solicitor General. It may be argued that Atty. show, however, that he failed to present clear, positive and Bounded on the North by property of Alejandro Marso; on the
Torres could have transmitted one of the two copies of absolute evidence 22 to overcome said presumption and to East by property of Ramon Bacor; on the South by property of
appellant's brief upon the Solicitor General, but such omission support his claim. Atty. Ricardo G. Mon and on the West by property of Librada
does not excuse Atienza's failure to serve a copy of his brief Guerrero. Containing an area of 6.0000 hectares, more or
directly on the Solicitor General. Atienza's claim is rooted in the March 9, 1932 decision of the less."
then Court of First Instance of Tayabas in Cadastral Case No.
On the part of the appellate court, its decision based solely on, 76, which was not given much weight by the court a quo, and However, on November 3, 1986, respondent applied with the
and even quoting verbatim from, the appellant's brief was for good reasons. Bureau of Lands in Isabela, Basilan province, for the award to
certainly arrived at in grave abuse of discretion. It denied him of the same parcel of land under free patent. On
appellee (petitioner herein) of the opportunity to be heard and Apart from his assertions before this Court, Atienza failed to November 17, 1986, petitioner filed a protest to such
to rebut Atienza's allegations, in rank disregard of its right to present proof that he or his predecessor-in-interest was one of application.
due process. Such violation of due process could have been the claimants who answered the petition filed by the then
rectified with the granting of petitioner's motion for Attorney-General in the said cadastral proceedings. The On February 7, 1989, the Regional Director of Lands rendered
reconsideration by the appellate court, 12 but even the door to document reflecting said cadastral decision, a xerox copy, a decision giving respondent a period of one hundred twenty
this recourse was slammed by the appellate court with the indicated the claimants simply as "Jose Abastillas et al." In (120) days to exercise the right to repurchase the land by
denial of petitioner's motion for extension of time to file motion support of that decision, Atienza presented a certification reimbursing petitioner of all expenses he incurred in the
for reconsideration in a resolution dated February 12, 1986, purportedly issued by someone from the Technical Reference purchase of the property in question, and held in abeyance
which ruling erroneously applied the Habaluyas doctrine. 13 Section of the Surveys Division, apparently of the Bureau of respondent's application for free patent.
Lands, stating that "Lot 5886 is a portion of Lot 5139 Pagbilao
Such denial notwithstanding, petitioner filed its motion for Cadastre," which evidence is, however, directly controverted On October 11, 1989, the Regional Director issued an order
reconsideration. Considering the clear allegations thereunder, by the sketch plan showing that the land in controversy is declaring that respondent had waived his right of repurchase,
the appellate court would have done well, in the interest of actually outside the alienable and disposable public lands, and rejected his application for free patent for lack of interest,
justice, not to blindly adhere to technical rules of procedure by although part of Lot 5139. and allowed petitioner to file a public land application for the
dismissing outright said motion. As we declared in Villareal v. subject land.
Court of Appeals: 14 The fact that Atienza acquired a title to the land is of no
moment, notwithstanding the indefeasibility of titles issued On May 8, 1990, the Regional Director ordered respondent to
. . . The requirements of due process are satisfied when the under the Torrens system. In Bornales v. Intermediate vacate the land in question, but respondent refused.
parties are afforded a fair and reasonable opportunity to Appellate Court, 23 we ruled that the indefeasibility of a
explain and air their side. The essence of due process is certificate of title cannot be invoked by one who procured the On July 24, 1990, petitioner filed with the Regional Trial Court,
simply the opportunity to be heard or as applied to same by means of fraud. The "fraud" contemplated by the law Basilan province, a complaint for recovery of ownership and
administrative proceedings, an opportunity to explain one's (Sec. 32, P.D. 1529) is actual and extrinsic, that is, "an possession with preliminary injunction of the subject parcel of
side or an opportunity to seek a reconsideration of the action intentional omission of fact required by law," 24 which in the land.
or ruling taken. (Emphasis supplied) case at bench consisted in the failure of Atienza to state that
the land sought to be registered still formed part of the In answer to the complaint, respondent alleged that the land
In view of the foregoing and the long-standing procedural rule unclassified forest lands. occupied by him belonged to the Republic of the Philippines,
that this Court may review the findings of facts of the Court of and that he had introduced improvements thereon such as
Appeals in the event that they may be contrary to those of the WHEREFORE, the decision appealed from is hereby coconut and other fruit trees.
trial court, 15 in order to attain substantial justice, the Court REVERSED and SET ASIDE and the decision of the court a
now reviews the facts of the case. quo dated July 27, 1981, is REINSTATED. After trial on the merits, on March 20, 1993, the trial court
rendered decision declaring petitioner the owner and
Under the Regalian Doctrine, all lands not otherwise clearly SO ORDERED. possessor of the subject parcel of land with all the
appearing to be privately-owned are presumed to belong to the improvements existing thereon, situated at Barangay Upper
State. Forest lands, like mineral or timber lands which are [G.R. No. 128017. January 20, 1999] Baas, municipality of Lantawan, province of Basilan, with an
public lands, are not subject to private ownership unless they RAMON ITURALDE, petitioner, vs. ALFREDO area of 3.1248 hectares, and ordering respondent to vacate
under the Constitution, become private properties. In the FALCASANTOS, respondent. the land in question, to pay petitioner the amount of ten
absence of such classification, the land remains unclassified PARDO, J.: thousand pesos (P10,000.00) as attorneys fee, the amount of
public land until released therefrom and rendered open to five thousand pesos (P5,000.00) as litigation expenses, and
disposition. 16 The case is an appeal via certiorari from a decision of the three hundred pesos (P300.00) as judicial cost.
Court of Appeals reversing that of the Regional Trial Court,
In our jurisdiction, the task of administering and disposing Branch 2, Basilan province, and dismissing petitioner's In due time, petitioner appealed the trial court's decision to the
lands of the public domain belongs to the Director of Lands, complaint for recovery of possession and ownership of a Court of Appeals.
and ultimately, the Secretary of Agriculture and Natural parcel of land with the improvements existing thereon, situated
Resources 17 (now the Secretary of Environment and Natural at Barangay Upper Baas, municipality of Lantawan, province of On December 20, 1996, the Court of Appeals rendered
Resources). 18 Classification of public lands is, thus, an Basilan, with an area of 7.1248 hectares. decision reversing the appealed decision, and entering a new
exclusive prerogative of the Executive Department through the judgment dismissing petitioner's complaint without prejudice to
Office of the President. 19 Courts have no authority to do so. The facts may be related as follows: any action that petitioner may take if the subject land was
20 declassified from forest land to alienable and disposable land
On October 17, 1986, petitioner acquired by purchase from the of the public domain.
Thus, in controversies involving the disposition of public heirs of Pedro Mana-ay a parcel of land located at Baas,
agricultural lands, the burden of overcoming the presumption Lantawan, Basilan Province, with an area of 6.0000 hectares, Hence, the present recourse.
of state ownership of lands of the public domain lies upon the more or less, more particularly described as follows:
12

Petitioner submits that the Court of Appeals erred in setting LA ROSA, represented by their father JOSE Y. DE LA recorded on January 2, 1931, in the office of the mining
aside the trial court's decision in his favor and dismissing the ROSA, respondents. recorder of Baguio. These claims were purchased from these
complaint because when the Director of Lands allowed locators on November 2, 1931, by Atok, which has since then
petitioner to file a public land application for said property, it G.R. No. L-44092 April 15, 1988 been in open, continuous and exclusive possession of the said
was equivalent to a declaration that said land was no longer ATOK-BIG WEDGE MINING COMPANY, petitioner, lots as evidenced by its annual assessment work on the
part of the public domain. vs. HON. COURT OF APPEALS, JOSE Y. DE LA ROSA, claims, such as the boring of tunnels, and its payment of
VICTORlA, BENJAMIN and EDUARDO, all surnamed DE annual taxes thereon. 9
We deny the petition. The Court of Appeals correctly held that LA ROSA, represented by their father, JOSE Y. DE LA
"the evidence is unrebutted that the subject land is within the ROSA, respondents. The location of the mineral claims was made in accordance
Forest Reserve Area as per L.C. Map No. 1557 certified on CRUZ, J.: with Section 21 of the Philippine Bill of 1902 which provided
August 13, 1951."[1] and, hence, not capable of private that:
appropriation and occupation.[2] The Regalian doctrine reserves to the State all natural wealth
that may be found in the bowels of the earth even if the land SEC. 21. All valuable mineral deposits in public lands in the
In Republic vs. Register of Deeds of Quezon, we held that where the discovery is made be private. 1 In the cases at bar, philippine Islands both surveyed and unsurveyed are hereby
"Forest lands, like mineral or timber lands which are public which have been consolidated because they pose a common declared to be free and open to exploration, occupation and
lands, are not subject to private ownership unless they under issue, this doctrine was not correctly applied. purchase and the land in which they are found to occupation
the Constitution, become private properties. In the absence of and purchase by the citizens of the United States, or of said
such classification, the land remains unclassified public land These cases arose from the application for registration of a islands.
until released therefrom and rendered open to disposition.[3] parcel of land filed on February 11, 1965, by Jose de la Rosa
on his own behalf and on behalf of his three children, Victoria, The Bureau of Forestry Development also interposed its
In Sunbeam Convenience Foods Inc. vs. Court of Appeals, we Benjamin and Eduardo. The land, situated in Tuding, Itogon, objection, arguing that the land sought to be registered was
said: Thus, before any land may be declassified from the forest Benguet Province, was divided into 9 lots and covered by plan covered by the Central Cordillera Forest Reserve under
group and converted into alienable or disposable land for Psu-225009. According to the application, Lots 1-5 were sold Proclamation No. 217 dated February 16, 1929. Moreover, by
agricultural or other purposes, there must be a positive act to Jose de la Rosa and Lots 6-9 to his children by Mamaya reason of its nature, it was not subject to alienation under the
from the government. Even rules on the confirmation of Balbalio and Jaime Alberto, respectively, in 1964. 2 Constitutions of 1935 and 1973. 10
imperfect titles do not apply unless and until the land classified
as forest land is released in an official proclamation to that The application was separately opposed by Benguet The trial court * denied the application, holding that the
effect so that it may form part of the disposable agricultural Consolidated, Inc. as to Lots 1-5, Atok Big Wedge Corporation, applicants had failed to prove their claim of possession and
lands of the public domain."[4] as to Portions of Lots 1-5 and all of Lots 6-9, and by the ownership of the land sought to be registered. 11 The
Republic of the Philippines, through the Bureau of Forestry applicants appealed to the respondent court, * which reversed
Hence, a positive act of the government is needed to Development, as to lots 1-9. 3 the trial court and recognized the claims of the applicant, but
declassify a forest land into alienable or disposable land for subject to the rights of Benguet and Atok respecting their
agricultural or other purposes.[5] In support of the application, both Balbalio and Alberto testified mining claims. 12 In other words, the Court of Appeals affirmed
that they had acquired the subject land by virtue of prescription the surface rights of the de la Rosas over the land while at the
And the rule is Possession of forest lands, however long, Balbalio claimed to have received Lots 1-5 from her father same time reserving the sub-surface rights of Benguet and
cannot ripen into private ownership.[6] shortly after the Liberation. She testified she was born in the Atok by virtue of their mining claims.
land, which was possessed by her parents under claim of
What is more, there is yet no award or grant to petitioner of the ownership. 4 Alberto said he received Lots 6-9 in 1961 from Both Benguet and Atok have appealed to this Court, invoking
land in question by free patent or other ways of acquisition of his mother, Bella Alberto, who declared that the land was their superior right of ownership. The Republic has filed its own
public land. Consequently, he can not lawfully claim to be the planted by Jaime and his predecessors-in-interest to bananas, petition for review and reiterates its argument that neither the
owner of the land in question. avocado, nangka and camote, and was enclosed with a private respondents nor the two mining companies have any
barbed-wire fence. She was corroborated by Felix Marcos, 67 valid claim to the land because it is not alienable and
WHEREFORE, the Court hereby AFFIRMS the appealed years old at the time, who recalled the earlier possession of registerable.
decision of the Court of Appeals in CA-G. R. CV No. 42306, the land by Alberto's father. 5 Balbalio presented her tax
dismissing the complaint of petitioner before the Regional Trial declaration in 1956 and the realty tax receipts from that year to It is true that the subject property was considered forest land
Court, Basilan province, in Civil Case No. 441-63. 1964, 6 Alberto his tax declaration in 1961 and the realty tax and included in the Central Cordillera Forest Reserve, but this
receipts from that year to 1964. 7 did not impair the rights already vested in Benguet and Atok at
No costs. that time. The Court of Appeals correctly declared that:
Benguet opposed on the ground that the June Bug mineral
SO ORDERED. claim covering Lots 1-5 was sold to it on September 22, 1934, There is no question that the 9 lots applied for are within the
by the successors-in-interest of James Kelly, who located the June Bug mineral claims of Benguet and the "Fredia and
G.R. No. L-43938 April 15, 1988 claim in September 1909 and recorded it on October 14, 1909. Emma" mineral claims of Atok. The June Bug mineral claim of
REPUBLIC OF THE PHILIPPINES (DIRECTOR OF FOREST From the date of its purchase, Benguet had been in actual, plaintiff Benguet was one of the 16 mining claims of James E.
DEVELOPMENT), petitioner, continuous and exclusive possession of the land in concept of Kelly, American and mining locator. He filed his declaration of
vs. HON. COURT OF APPEALS (THIRD DIVISION) and owner, as evidenced by its construction of adits, its affidavits of the location of the June Bug mineral and the same was
JOSE Y. DE LA ROSA, respondents. annual assessment, its geological mappings, geological recorded in the Mining Recorder's Office on October 14, 1909.
samplings and trench side cuts, and its payment of taxes on All of the Kelly claims ha subsequently been acquired by
G.R. No. L-44081 April 15, 1988 the land. 8 Benguet Consolidated, Inc. Benguet's evidence is that it had
BENGUET CONSOLIDATED, INC., petitioner, made improvements on the June Bug mineral claim consisting
vs. HON. COURT OF APPEALS, JOSE Y. DE LA ROSA, For its part, Atok alleged that a portion of Lots 1-5 and all of of mine tunnels prior to 1935. It had submitted the required
VICTORIA, BENJAMIN and EDUARDO, all surnamed DE Lots 6-9 were covered by the Emma and Fredia mineral claims affidavit of annual assessment. After World War II, Benguet
located by Harrison and Reynolds on December 25, 1930, and introduced improvements on mineral claim June Bug, and also
13

conducted geological mappings, geological sampling and It is of no importance whether Benguet and Atok had secured respondents aver, by acquisitive prescription. However, the
trench side cuts. In 1948, Benguet redeclared the "June Bug" a patent for as held in the Gold Creek Mining Corp. Case, for method invoked by the de la Rosas is not available in the case
for taxation and had religiously paid the taxes. all physical purposes of ownership, the owner is not required to at bar, for two reasons.
secure a patent as long as he complies with the provisions of
The Emma and Fredia claims were two of the several claims of the mining laws; his possessory right, for all practical purposes First, the trial court found that the evidence of open,
Harrison registered in 1931, and which Atok representatives of ownership, is as good as though secured by patent. continuous, adverse and exclusive possession submitted by
acquired. Portions of Lots 1 to 5 and all of Lots 6 to 9 are the applicants was insufficient to support their claim of
within the Emma and Fredia mineral claims of Atok Big Wedge We agree likewise with the oppositors that having complied ownership. They themselves had acquired the land only in
Mining Company. with all the requirements of the mining laws, the claims were 1964 and applied for its registration in 1965, relying on the
removed from the public domain, and not even the government earlier alleged possession of their predecessors-in-interest. 16
The June Bug mineral claim of Benguet and the Fredia and of the Philippines can take away this right from them. The The trial judge, who had the opportunity to consider the
Emma mineral claims of Atok having been perfected prior to reason is obvious. Having become the private properties of the evidence first-hand and observe the demeanor of the
the approval of the Constitution of the Philippines of 1935, they oppositors, they cannot be deprived thereof without due witnesses and test their credibility was not convinced. We
were removed from the public domain and had become private process of law. 13 defer to his judgment in the absence of a showing that it was
properties of Benguet and Atok. reached with grave abuse of discretion or without sufficient
Such rights were not affected either by the stricture in the basis. 17
It is not disputed that the location of the mining claim under Commonwealth Constitution against the alienation of all lands
consideration was perfected prior to November 15, 1935, when of the public domain except those agricultural in nature for this Second, even if it be assumed that the predecessors-in-
the Government of the Commonwealth was inaugurated; and was made subject to existing rights. Thus, in its Article XIII, interest of the de la Rosas had really been in possession of the
according to the laws existing at that time, as construed and Section 1, it was categorically provided that: subject property, their possession was not in the concept of
applied by this court in McDaniel v. Apacible and Cuisia (42 owner of the mining claim but of the property as agricultural
Phil. 749), a valid location of a mining claim segregated the SEC. 1. All agricultural, timber and mineral lands of the land, which it was not. The property was mineral land, and they
area from the public domain. Said the court in that case: The public domain, waters, minerals, coal, petroleum and other were claiming it as agricultural land. They were not disputing
moment the locator discovered a valuable mineral deposit on mineral oils, all forces of potential energy and other natural the lights of the mining locators nor were they seeking to oust
the lands located, and perfected his location in accordance resources of the Philipppines belong to the State, and their them as such and to replace them in the mining of the land. In
with law, the power of the United States Government to disposition, exploitation, development, or utilization shall be fact, Balbalio testified that she was aware of the diggings being
deprive him of the exclusive right to the possession and limited to citizens of the Philippines or to corporations or undertaken "down below" 18 but she did not mind, much less
enjoyment of the located claim was gone, the lands had associations at least 60% of the capital of which is owned by protest, the same although she claimed to be the owner of the
become mineral lands and they were exempted from lands that such citizens, subject to any existing right, grant, lease or said land.
could be granted to any other person. The reservations of concession at the time of the inauguration of the government
public lands cannot be made so as to include prior mineral established under this Constitution. Natural resources with the The Court of Appeals justified this by saying there is "no
perfected locations; and, of course, if a valid mining location is exception of public agricultural lands, shall not be alienated, conflict of interest" between the owners of the surface rights
made upon public lands afterwards included in a reservation, and no license, concession, or lease for the exploitation, and the owners of the sub-surface rights. This is rather
such inclusion or reservation does not affect the validity of the development or utilization of any of the natural resources shall doctrine, for it is a well-known principle that the owner of piece
former location. By such location and perfection, the land be granted for a period exceeding 25 years, except as to water of land has rights not only to its surface but also to everything
located is segregated from the public domain even as against rights for irrigation, water supply, fisheries, or industrial uses underneath and the airspace above it up to a reasonable
the Government. (Union Oil Co. v. Smith, 249 U.S. 337; Van other than the development of water power, in which case height. 19 Under the aforesaid ruling, the land is classified as
Mess v. Roonet, 160 Cal. 131; 27 Cyc. 546). beneficial use may be the measure and the limit of the grant. mineral underneath and agricultural on the surface, subject to
separate claims of title. This is also difficult to understand,
"The legal effect of a valid location of a mining claim is not only Implementing this provision, Act No. 4268, approved on especially in its practical application.
to segregate the area from the public domain, but to grant to November 8, 1935, declared:
the locator the beneficial ownership of the claim and the right Under the theory of the respondent court, the surface owner
to a patent therefor upon compliance with the terms and Any provision of existing laws, executive order, proclamation to will be planting on the land while the mining locator will be
conditions prescribed by law. Where there is a valid location of the contrary notwithstanding, all locations of mining claim boring tunnels underneath. The farmer cannot dig a well
a mining claim, the area becomes segregated from the public made prior to February 8, 1935 within lands set apart as forest because he may interfere with the operations below and the
domain and the property of the locator." (St. Louis Mining & reserve under Sec. 1826 of the Revised Administrative Code miner cannot blast a tunnel lest he destroy the crops above.
Milling Co. v. Montana Mining Co., 171 U.S. 650; 655; 43 Law which would be valid and subsisting location except to the How deep can the farmer, and how high can the miner, go
ed., 320, 322.) "When a location of a mining claim is perfected existence of said reserve are hereby declared to be valid and without encroaching on each other's rights? Where is the
it has the effect of a grant by the United States of the right of subsisting locations as of the date of their respective locations. dividing line between the surface and the sub-surface rights?
present and exclusive possession, with the right to the
exclusive enjoyment of all the surface ground as well as of all The perfection of the mining claim converted the property to The Court feels that the rights over the land are indivisible and
the minerals within the lines of the claim, except as limited by mineral land and under the laws then in force removed it from that the land itself cannot be half agricultural and half mineral.
the extralateral right of adjoining locators; and this is the the public domain. 14 By such act, the locators acquired The classification must be categorical; the land must be either
locator's right before as well as after the issuance of the exclusive rights over the land, against even the government, completely mineral or completely agricultural. In the instant
patent. While a lode locator acquires a vested property right by without need of any further act such as the purchase of the case, as already observed, the land which was originally
virtue of his location made in compliance with the mining laws, land or the obtention of a patent over it. 15 As the land had classified as forest land ceased to be so and became mineral
the fee remains in the government until patent issues."(18 become the private property of the locators, they had the right — and completely mineral — once the mining claims were
R.C.L. 1152) (Gold Creek Mining Corporation v. Hon. Eulogio to transfer the same, as they did, to Benguet and Atok. perfected. 20 As long as mining operations were being
Rodriguez, Sec. of Agriculture and Commerce, and Quirico undertaken thereon, or underneath, it did not cease to be so
Abadilla, Director of the Bureau of Mines, 66 Phil. 259, 265- It is true, as the Court of Appeals observed, that such private and become agricultural, even if only partly so, because it was
266) property was subject to the "vicissitudes of ownership," or even enclosed with a fence and was cultivated by those who were
to forfeiture by non-user or abandonment or, as the private unlawfully occupying the surface.
14

What must have misled the respondent court is WHEREFORE, the decision of the respondent court dated Norte issued a new certificate of title under Transfer Certificate
Commonwealth Act No. 137, providing as follows: April 30, 1976, is SET ASIDE and that of the trial court dated of Title (TCT) No. (T-347)-2927 under the names of the heirs
March 11, 1969, is REINSTATED, without any pronouncement on 13 January 1954.
Sec. 3. All mineral lands of the public domain and minerals as to costs.
belong to the State, and their disposition, exploitation, On 30 July 1992, Concepcion caused the survey and the
development or utilization, shall be limited to citizens of the SO ORDERED. subdivision of the entire property including the portion
Philippines, or to corporations, or associations, at least 60% of occupied by PUMCO-SDA.8 It was at this point that
the capital of which is owned by such citizens, subject to any G.R. No. 195990, August 05, 2015 respondents brought to the attention of Concepcion that the
existing right, grant, lease or concession at the time of the HEIRS OF RAFAEL GOZO REPRESENTED BY CASTILLO 5,000 square-meter portion of the property is already owned by
inauguration of government established under the Constitution. GOZO AND RAFAEL GOZO, JR., Petitioners, v. respondent PUMCO-SDA in view of the Deed of Donation she
PHILIPPINE UNION MISSION CORPORATION OF THE executed together with her husband in their favor in 1937.
SEC. 4. The ownership of, and the right to the use of land for SEVENTH DAY ADVENTIST CHURCH (PUMCO), SOUTH When Concepcion, however, verified the matter with the
agricultural, industrial, commercial, residential, or for any PHILIPPINE UNION MISSION OF SDA (SPUMCO) AND Register Deeds, it appeared that the donation was not
purpose other than mining does not include the ownership of, SEVENTH DAY ADVENTIST CHURCH AT SIMPAK, LALA, annotated in the title. The absence of annotation of the so-
nor the right to extract or utilize, the minerals which may be LANAO DEL NORTE REPRESENTED BY BETTY PEREZ , called encumbrance in the title prompted petitioners not to
found on or under the surface. Respondents. recognize the donation claimed by the respondents. The
PEREZ, J.: matter was left unresolved until Concepcion died and the rest
SEC. 5. The ownership of, and the right to extract and utilize, of the owners continued to pursue their claims to recover the
the minerals included within all areas for which public This is a Petition for Review on Certiorari1 filed by petitioners subject property from the respondents.
agricultural land patents are granted are excluded and Heirs of Rafael Gozo seeking to reverse and set aside the 10
excepted from all such patents. November 2010 Decision2 of the Court of Appeals and its 14 A compromise was initially reached by the parties wherein the
February 2011 Resolution3 in CA-G.R. CV No. 00188. The petitioners were allowed by respondents to harvest from the
SEC. 6. The ownership of, and the right to extract and utilize, assailed decision and resolution reversed the 30 June 2004 coconut trees planted on the subject property but a
the minerals included within all areas for which Torrens titles Decision of the Regional Trial Court (RTC) of Kapatagan, misunderstanding ensued causing respondents to file a case
are granted are excluded and excepted from all such titles. Lanao del Norte and held that the action for nullification and for qualified theft against the petitioners.
recovery of possession filed by the petitioners is already,
This is an application of the Regalian doctrine which, as its barred by laches. The dispositive portion of the Court of On 19 June 2000 or around six decades after the Deed of
name implies, is intended for the benefit of the State, not of Appeals Decision reads:chanRoblesvirtualLawlibrary Donation was executed, petitioners filed an action for
private persons. The rule simply reserves to the State all Declaration of Nullity of Document, Recovery of Possession
minerals that may be found in public and even private land ACCORDINGLY, the Decision dated 30 June 2004 of the court and Ownership with Damages against PUMCO-SDA before
devoted to "agricultural, industrial, commercial, residential or a quo is REVERSED and SET ASIDE. The South Philippine the RTC of Kapatagan, Lanao del Norte.9 In their-Complaint
(for) any purpose other than mining." Thus, if a person is the Union Mission of the Seventh Day Adventist Church remains docketed as Civil Case No. 21-201, petitioners claimed that the
owner of agricultural land in which minerals are discovered, his the absolute owner of the donated possession of PUMCO-SDA on the subject property was
ownership of such land does not give him the right to extract or property.4ChanRoblesVirtualawlibrary merely tolerated by petitioners and therefore could not ripen
utilize the said minerals without the permission of the State to ChanRoblesVirtualawlibrary into ownership.10 In addition, petitioners argued that the
which such minerals belong. The Facts signatures of the Spouses Gozo were forged underscoring the
stark contrast between the genuine signatures of their parents
The flaw in the reasoning of the respondent court is in Petitioners claim that they are the heirs of the Spouses Rafael from the ones appearing in the deed.11 Finally, petitioners
supposing that the rights over the land could be used for both and Concepcion Gozo (Spouses Gozo) who, before their averred that granting for the sake of argument that the said
mining and non-mining purposes simultaneously. The correct death, were the original owners of a parcel of land with an area signatures were genuine, the deed of donation will remain
interpretation is that once minerals are discovered in the land, 236,638 square meters located in Sitio Simpak, Brgy. Lala, invalid for lack of acceptance which is an essential requisite for
whatever the use to which it is being devoted at the time, such Municipality of Kolambugan, Lanao del Norte. The a valid contract of donation.12
use may be discontinued by the State to enable it to extract the respondents claim that they own a 5,000 square-meter portion
minerals therein in the exercise of its sovereign prerogative. of the property. The assertion is based on the 28 February For their part, respondents insisted on the validity of the
The land is thus converted to mineral land and may not be 1937 Deed of Donation5 in favor of respondent Philippine donation and on the genuineness of the signatures of the
used by any private party, including the registered owner Union Mission Corporation of the Seventh Day Adventist donors who had voluntarily parted with their property as faithful
thereof, for any other purpose that will impede the mining (PUMCO-SDA). Respondents took possession of the subject devotees of the church for the pursuit of social and religious
operations to be undertaken therein, For the loss sustained by property by introducing improvements thereon through the ends.13 They further contended that from the moment the
such owner, he is of course entitled to just compensation construction of a church building, and later on, an elementary Spouses Gozo delivered the subject property to respondents in
under the Mining Laws or in appropriate expropriation school. On the date the Deed of Donation is executed in 1937, 1937, they were already in open, public, continuous and
proceedings. 21 the Spouses Gozo were not the registered owners of the adverse possession thereof in the concept of an owner.14 A
property yet although they were the lawful possessors thereof. considerable improvement was claimed to have been
Our holding is that Benguet and Atok have exclusive rights to It was only on 5 October 1953 that the Original Certificate of introduced into the property in the form of church and school
the property in question by virtue of their respective mining Title (OCT) No. P-642 covering the entire property was issued buildings.15 The argument of the petitioners, therefore, that
claims which they validly acquired before the Constitution of in the name of Rafael Gozo (Rafael) married to Concepcion the donation was invalid for lack of acceptance, a question
1935 prohibited the alienation of all lands of the public domain Gozo (Concepcion) pursuant to the Homestead Patent granted which came 63 years after it was executed, is already barred
except agricultural lands, subject to vested rights existing at by the President of the Philippines on 22 August 1953.6 by laches.
the time of its adoption. The land was not and could not have
been transferred to the private respondents by virtue of In view of Rafael's prior death, however, his heirs, Concepcion, After the pre-trial conference, trial on the merits ensued. Both
acquisitive prescription, nor could its use be shared and their six children, namely, Abnera, Benia, Castillo, Dilbert, parties adduced documentary and testimonial evidence to
simultaneously by them and the mining companies for Filipinas and Grace caused the extrajudicial partition of the support their respective positions.
agricultural and mineral purposes. property. Accordingly, the Register of Deeds of Lanao del
15

On 30 June 2004, the RTC rendered a Decision16 in favor of dispose of the land which, by all intents and purposes, belongs SEC. 7. For the purposes of the administration and disposition
the petitioners thereby declaring that they are the rightful to the State. of alienable or disposable public lands, the Batasang
owners of the subject property since the contract of donation Pambansa or the President, upon recommendation by the
which purportedly transferred the. ownership of the subject Under the Regalian doctrine, which is embodied in Article XII, Secretary of Natural Resources, may from time to time declare
property to PUMCO-SDA is void for lack of acceptance. In Section 2 of our Constitution, all lands of the public domain what public lands are open to disposition or concession under
upholding the right of the petitioners to the land, the court a belong to the State, which is the source of any asserted right to this Act.
quo held that an action or defense for the declaration of nullity any ownership of land. All lands not appearing to be clearly
of a contract does not prescribe. Anent the claim that within private ownership are presumed to belong to the State. xxxx
petitioners slept on their rights, the RTC adjudged that the Accordingly, public lands not shown to have been reclassified
equitable doctrine of laches is inapplicable in the case at bar or released as alienable agricultural land or alienated to a SEC. 8. Only those lands shall be declared open to disposition
because the action of the registered owners to recover private person by the State remain part of the inalienable or concession which have been officially delimited and
possession is based on Torrens title which cannot be barred public domain.21 classified and, when practicable, surveyed, and which have not
by laches. The RTC disposed in this been reserved for public or quasi-public uses, nor appropriated
wise:chanRoblesvirtualLawlibrary The classification of public lands is an exclusive prerogative of by the Government, nor in any manner become private
WHEREFORE, in view of the foregoing consideration, the executive department of the government and not the property, nor those on which a private right authorized and
judgment is hereby rendered in favor of the [petitioners], to wit: Courts. In the absence of such classification, the land remains recognized by this Act or any other valid law may be claimed,
as an unclassified land until it is released therefrom and or which, having been reserved or appropriated, have ceased
(1) rendered open to disposition. This is in consonance with the to be so. However, the President may, for reasons of public
Declaring the 5,000 square meter portion of the land covered Regalian doctrine that all lands of the public domain belong to interest, declare lands of the public domain open to disposition
by TCT [No.] (T-347)-292 part of the common property of the the State and that the State is the source of any asserted right before the same have had their boundaries established or
[petitioners]; and to ownership in land and charged with the conservation of such been surveyed, or may, for the same reason, suspend their
(2) patrimony.22 concession or disposition until they are again declared open to
Declaring the Deed of Donation as void. concession or disposition by proclamation duly published or by
All lands not appearing to be clearly within private ownership Act of the Congress.
The [petitioners], however, are not entitled to damages, are presumed to belong to the State. Accordingly, all public
attorney's fees and cost of litigation prayed lands not shown to have been reclassified or released as SEC. 9. For the purpose of their administration and disposition,
for.17ChanRoblesVirtualawlibrary alienable agricultural land or alienated to a private person by the lands of the public domain alienable or open to disposition
On appeal, the Court of Appeals reversed the RTC Decision18 the State remain part of the alienable public domain. As shall be classified, according to the use or purposes to which
and ordered the dismissal of petitioners' complaint on the already well-settled in jurisprudence, no public land can be such lands are destined, as
ground of laches. The appellate court opined that petitioners acquired by private persons without any grant, express or follows:chanRoblesvirtualLawlibrary
failed to assert their rights over the land for more than 60 implied, from the government; and it is indispensable that the
years, thus, laches had set in. Even if petitioners were the person claiming title to public land should show that his title (a) Agricultural;cralawlawlibrary
registered owners of the land in dispute, laches would still bar was acquired from the State or any other mode of acquisition (b) Residential, commercial, industrial, or for similar productive
them from recovering possession of the same. recognized by law. To prove that the land subject of an purposes;cralawlawlibrary
application for registration is alienable, the applicant must (c) Educational, charitable, or other similar purposes; and
The Motion for Reconsideration filed by the petitioners was establish the existence of a positive act of the government (d) Reservations for townsites and for public and quasi-public
likewise denied by the appellate court in a Resolution19 dated such as a presidential proclamation or an executive order, an uses.
14 February 2011. administrative action, investigation reports of Bureau of Lands The President, upon recommendation by the Secretary of
investigators, and a legislative act or a statute. The applicant Agriculture and Natural Resources, shall from time to time
Petitioners are now before this Court via this instant Petition for may also secure a certification from the Government that the make the classifications provided for in this section, and may,
Review on Certiorari seeking the reversal of the Court of land applied for is alienable and disposable.23 at any time and in a similar manner, transfer lands from one
Appeals Decision and Resolution on the sole ground class to another.25cralawredChanRoblesVirtualawlibrary
that:chanRoblesvirtualLawlibrary Commonwealth Act No. 141, also known as the Public Land By virtue of Presidential Decree No. 705, otherwise known as
THE HONORABLE COURT OF APPEALS ERRED IN Act, as amended by Presidential Decree No. 1073, remains to the Revised Forestry Code, the President delegated to the
REVERSING AND SETTING ASIDE THE DECISION OF THE this day the existing general law governing the classification DENR Secretary the power to determine which of the
COURT A QUO ON THE GROUND OF and disposition of lands of the public domain, other than timber unclassified lands of the public domain are (1) needed for
LACHES.20ChanRoblesVirtualawlibrary and mineral lands. The following provisions under Title I, forest purposes and declare them as permanent forest to form
ChanRoblesVirtualawlibrary Chapter II of the Public Land Act, as amended, is very specific part of the forest reserves; and (2) not needed for forest
The Court's Ruling on how lands of the public domain become alienable or purposes and declare them as alienable and disposable
disposable:24 lands.26
While the opposing parties center their arguments and counter SEC. 6. The President, upon the recommendation of the
arguments on the timeliness of raising the question of the Secretary of Agriculture and Natural Resources, shall from Per the Public Land Act, alienable and disposable public lands
validity of' donation, a careful scrutiny of the records, however, time to time classify the lands of the public domain suitable for agricultural purposes can be disposed of only as
reveals a significant fact that at the time the Deed of Donation into:chanRoblesvirtualLawlibrary follows:chanRoblesvirtualLawlibrary
was executed by the Spouses Gozo on 28 February 1937, the (a) Alienable or disposable, 1. For homestead settlement;cralawlawlibrary
subject property was part of the inalienable public domain. It (b) Timber, and 2. By sale;cralawlawlibrary
was only almost after two decades later or on 5 October 1953 (c) Mineral lands,ChanRoblesVirtualawlibrary 3. By lease; and
that the State ceded its right over the land in favor of the and may at any time and in a like manner transfer such lands 4. By confirmation of imperfect or incomplete titles:
Spouses Gozo by granting their patent application and issuing from one class to another, for the purposes of their (a) By judicial legalization;cralawlawlibrary
an original certificate of title in their favor. Prior to such administration and disposition. (b) By administrative legalization (free patent).27
conferment of title, the Spouses Gozo possessed no right to Homestead over alienable and disposable public agricultural
land is granted after compliance by an applicant with the
16

conditions and requirements laid down under Title II, Chapter hectares in 1936 including the disputed 12.8081 hectares for at Clearly from the facts, the petitioners asserted their rights
IV of the Public Land Act, the most basic of which are quoted the time this 12.8081-hectare lot had already been severed repeatedly; it was the respondents who kept silent all
below:chanRoblesvirtualLawlibrary from the mass disposable public lands by Proclamation No. 85 throughout about the supposed donee's rights.
SEC. 12. Any citizen of the Philippines over the age of and excluded from the Sales Award. Impoverishment of
eighteen years, or the head of a family, who does not own Eugenio's asset as a result of such donation is therefore WHEREFORE, premises considered, the instant petition is
more than twenty-four hectares of land in the Philippines or farfetched. (Emphasis supplied)ChanRoblesVirtualawlibrary hereby GRANTED. The assailed Decision dated 10 November
has not had the benefit of any gratuitous allotment of more It is beyond question that at the time the gratuitous transfer 2010 and Resolution dated 14 February 2011 of the Court of
than twenty-four hectares of land since the occupation of the was effected by the Spouses Gozo on 28 February 1937, the Appeals in CA-G.R. CV No. 00188 are hereby REVERSED
Philippines by the United States, may enter a homestead of subject property was part of the public domain and is outside and SET ASIDE.
not exceeding twenty-four hectares of agricultural land of the the commerce of man. It was only on 5 October 1953 that the
public domain. ownership of the property was vested by the State to the SO ORDERED.
Spouses Gozo by virtue of its issuance of the OCT pursuant to
SEC. 13. Upon the filing of an application for a homestead, the the Homestead Patent granted by the President of the G.R. No. 186603
Director of Lands, if he finds that the application should be Philippines on 22 August 1953. Hence, the donation of the REPUBLIC OF THE PHILIPPINES, represented by the
approved, shall do so and authorize the applicant to take subject property which took place before 5 October 1953 is null REGIONAL EXECUTIVE DIRECTOR, DENR, REGION VI,
possession of the land upon the payment of five pesos, and void from the very start.33 ILOILO CITY, Petitioners,
Philippine currency, as entry fee. Within six months from and vs. VALENTINA REGISTER OF PROVINCE OCCIDENTAL,
after the date of the approval of the application, the applicant As a void contract, the Deed of Donation produces no legal CALISTON, DIOSCORO ESCARDA, ESPINOSA, DEEDS OF
shall begin to work the homestead, otherwise he shall lose his effect whatsoever. Quod nullum est, nullum producit THE OF NEG ROS LEONILA and & SPOUSES ESTRELLA,
prior right to the land. effectum34 That which is a nullity produces no effect.35 Respondents
Logically, it could not have transferred title to the subject JARDELEZA, J.:
SEC. 14. No certificate shall be given or patent issued for the property from the Spouses Gozo to PUMCO-SDA and there
land applied for until at least one-fifth of the land has been can be no basis for the church's demand for the issuance of This is a petition for review on certiorari1 seeking to nullify the
improved and cultivated. The period within which the land shall title under its name. Neither does the church have the right to Court of Appeals' (CA) July 25, 2008 Decision2 and February
be cultivated shall not be less than one nor more than five subsequently dispose the property nor invoke acquisitive 4, 2009 Resolution3 in CA-G.R. CV No. 00421. The CA
years, from and after the date of the approval of the prescription to justify its occupation. A void contract is not modified the May 12, 2004 Decision4 of the Regional Trial
application. The applicant shall, within the said period, notify susceptible to ratification, and the action for the declaration of Court (RTC), Branch 61 of Kabankalan City, Negros
the Director of Lands as soon as he is ready to acquire the absolute nullity of such contract is imprescriptible.36 Occidental, and dismissed the reversion case filed by the
title. If at the date of such notice, the applicant shall prove to Republic of the Philippines (State) against respondents
the satisfaction of the Director of Lands, that he has resided The lack of respondents' right over the property was confirmed Valentina Espinosa and her successor-in-interest, Leonila B.
continuously for at least one year in the municipality in which when the Spouses Gozo had the entire property, including the Caliston, to wit: WHEREFORE, the appeal is GRANTED. The
the land is located, or in a municipality adjacent to the same portion occupied by the church, surveyed and patented, and Decision dated May 12, 2004 and Order dated July 16, 2004
and has cultivated at least one-fifth of the land continuously covered by their homestead patent. Further, after a certificate are hereby modified upholding the validity of Original
since the approval of the application, and shall make affidavit of title was issued under their names, the Spouses Gozo did Certificate of Title No. 191-N and Transfer Certificate of Title
that no part of said land has been alienated or encumbered, not effect the annotation thereon of the supposed donation. No. 91117, respectively, issued in the names of Valentina
and that he has complied with all the requirements of this Act, Registration is the operative act that gives validity to the Espinosa and Leonila Caliston. The award of damages,
then, upon the payment of five pesos, as final fee, he shall be transfer or creates a lien upon the land.37 Indeed it has been attorney's fees and expenses of litigation in favor of Leonila
entitled to a patent.28ChanRoblesVirtualawlibrary ruled that where there was nothing in the certificate of title to Caliston is affirmed.
It is clear under the law that before compliance with the indicate any cloud or vice in the ownership of the property, or
foregoing conditions and requirements the applicant has no any encumbrance thereon, the purchaser is not required to SO ORDERED.5
right over the land subject of the patent and therefore cannot explore farther than what the Torrens title upon its face
dispose the same even if such disposal was made gratuitously. indicates in quest for any hidden defect or inchoate right that On October 26, 1955, Cadastral Decree No. N-31626 was
It is an established principle that no one can give what one may subsequently defeat his right thereto.38 If the rule were issued to Valentina Espinosa (Espinosa) in Cadastral Case
does not have, nemo dat quod non habet.29 It is true that otherwise, the efficacy and conclusiveness of the certificate of No. 39, L.R.C. Cadastral Record No. 980. It covered a 28,880-
gratuitous disposal in donation may consist of a thing or a right title which the Torrens system seeks to insure would entirely square meter lot located at Lot No. 3599 of Cadastral Record
but the term right must be understood in a "proprietary" sense be futile and nugatory.39 The public shall then be denied of its No. 980, Poblacion, Sipalay City, Negros Occidental
over which the possessor has jus disponendi.30 This is foremost motivation for respecting and observing the Land (property). By virtue of the decree, Original Certificate of Title
because in true donations there results a consequent Registration Act.40 (OCT) No. 191-N was issued on October 15, 1962 in the name
impoverishment of the donor or diminution of his assets.31 In ofEspinosa.6 On June 17, 1976, Espinosa sold the property to
Republic v. Court of Appeals,32 the Court declared the Just as significantly, the homestead application of the Spouses Leonila B. Caliston (Caliston), who was later issued Transfer
contract of donation, executed by the donor who has no Gozo over the entire area of the property including that Certificate of Title (TCT) No. T- 911177 on June 29, 1976.8
proprietary right over the object of the contract, null and void, occupied by respondents and the issuance in their favor of the
viz:chanRoblesvirtualLawlibrary corresponding title without any complaint or objection from the On January 13, 2003, the State, represented by the Regional
Even on the gratuitous assumption that a donation of the respondents, remove the case of the petitioners from the Executive Director of the Department of Environment and
military "camp site" was executed between Eugenio de Jesus operation of the doctrine of laches. Natural Resources (DENR), Region VI, Iloilo City, through the
and Serafin Marabut, such donation would anyway be void Office of the Solicitor General (OSG), filed a Complaint9 for
because Eugenio de Jesus held no dominical rights over the And, further than the issuance of an original title, the entire annulment of title and/or reversion of land with the RTC,
site when it was allegedly donated by him in 1936. In that year, property was made subject of an extrajudicial partition of the Branch 61 of Kabankalan City, Negros Occidental. The State
Proclamation No. 85. of President Quezon already withdrew property by the Gozo heirs resulting in the issuance of TCTs in claimed that the property is inalienable public land because it
the area from sale or settlement and reserved it for military their names in 1954. Again, in no instance during the partition fell within a timberland area indicated under Project No. 27-C,
purposes, x x x Eugenio de Jesus cannot be said to be did the respondents make known their claim over the property. Block C per Land Classification (LC) Map No. 2978, as
possessed of that "proprietary" right over the whole 33 certified by the Director of Forestry on January 17, 1986.10
17

The spouses Dioscoro and Estrella Escarda (spouses Caliston' s motion for reconsideration 19 was denied in an In land registration proceedings, the applicant has the burden
Escarda) intervened, 11 alleging that they have been Order20 dated July 16, 2004. On August 5, 2004, Caliston filed of overcoming the presumption of State ownership. It must
occupying the property since 1976 on the belief that it belongs a Notice of Appeal21 with the RTC. On the other hand, the establish, through incontrovertible evidence, that the land
to the State. 12 They prayed that Caliston be ordered to cease spouses Escarda did not file a notice of appeal. Records were sought to be registered is alienable or disposable based on a
and desist from ejecting them. 13 then forwarded to the CA, where proceedings ensued. positive act of the government. 30 Since cadastral proceedings
are governed by the usual rules of practice, procedure, and
In answer, Caliston countered that the property is not There, Caliston argued that the trial court improperly relied evidence, a cadastral decree and a certificate of title are
timberland. Invoking laches and prescription, she argued that upon LC Map No. 2978, which was prepared long after the issued only after the applicant proves all the requisite
her title was issued earlier in 1962, while the map shows that property was alienated and awarded to Espinosa, her jurisdictional facts-that they are entitled to the claimed lot, that
the property was classified only in 1986.14 Caliston also predecessor-in-interest. The map, the admissibility and all parties are heard, and that evidence is considered.31 As
claimed that the spouses Escarda lacked the capacity or genuineness of which have yet to be proved, cannot be used such, the cadastral decree is a judgment which adjudicates
personality to intervene because only the State may initiate an to defeat the cadastral proceedings presumed to have been ownership after proving these jurisdictional facts.32
action for reversion. She also alleged that the spouses regularly conducted. Even assuming the map can be
Escarda cannot claim a better right as against her because considered, Caliston claims that her property is situated in an Here, it is undisputed that Espinosa was granted a cadastral
she merely tolerated their occupancy of the property until their area indicated as alienable and disposable. She also reiterated decree and was subsequently issued OCT No. 191-N, the
refusal to vacate it. 15 As counterclaim, Caliston claimed for her defenses of laches and prescription. 22 predecessor title of Caliston's TCT No. 91117. Having been
moral and exemplary damages, attorney's fees and litigation granted a decree in a cadastral proceeding, Espinosa can be
expenses against the spouses Escarda for the baseless and For its part, the State argued that the lower court did not err in presumed to have overcome the presumption that the land
malicious complaint. 16 relying upon LC Map No. 2978 though it was prepared only in sought to be registered forms part of the public domain. 33
1986. According to the State, forest lands are incapable of This means that Espinosa, as the applicant, was able to prove
The RTC rendered a Decision17 dated May 12, 2004. Relying private appropriation and possession, however long; by incontrovertible evidence that the property is alienable and
on LC Map No. 2978, the trial court ruled in favor of the State prescription does not run against the government. 23 disposable property in the cadastral proceedings.
and ordered the reversion of the property to the mass of the
public domain, viz.: The CA rendered a Decision24 dated July 25, 2008 modifying This is not to say, however, that the State has no remedy to
the RTC Decision. It upheld the validity of OCT No. 191-N and recover the property if indeed it is part of the inalienable lands
WHEREFORE, premises considered, judgment 1s hereby TCT No. 91117 issued in the names of Espinosa and Caliston, of the public domain. The State may still do so through an
rendered as follows: respectively, and affirmed the award of damages, attorney's action for reversion, as in the present case.
fees, and expenses of litigation in favor of Caliston.
1. Declaring Original Certificate of Title No.191-N in the name Reversion is the remedy where the State, pursuant to the
of Valentina Espinosa and all its derivative titles, such as: TCT The CA found that the State failed to prove fraud or Regalian doctrine, seeks to revert land back to the mass of the
No. T-91117 in the name of Leonila Caliston, null and void ab misrepresentation on the part of Espinosa when she was public domain. 34 It is proper when public land is fraudulently
initio; issued OCT No. 191-N. It further ruled that the State failed to awarded and disposed of to private individuals or
prove that the property is forest land. The lone piece of corporations.35 There are also instances when we granted
2. Ordering defendants to surrender the owner's duplicate evidence consisting of LC Map No. 2978, certified by the reversion on grounds other than fraud, such as when a
copy of OCT No. 191-N and TCT N[o]. T-91117 to defendant Director of Forestry on January 17, 1986, was not "person obtains a title under the Public Land Act which
Register of Deeds for the Province of Negros Occidental and authenticated pursuant to Section 24, 25 Rule 132 of the Rules includes, by oversight, lands which cannot be registered under
the latter to cancel said titles and all their derivative titles, if of Court. It noted that the parties stipulated only as to the the Torrens system, or when the Director of Lands did not
any; existence of the map, but not as to its genuineness or the have jurisdiction over the same because it is of the public
truthfulness of its content. Assuming that the map is admitted domain."36
3. Ordering the reversion of the land covered by the aforesaid in evidence, Espinosa's rights over the property, which
patent and title to the mass of the public domain under the accrued in 1962; should not be prejudiced by a subsequent In this case, the State, through the Solicitor General, alleges
administration and disposition of the Director of Forestry (now classification by the State done in 1986, or after 24 years. 26 neither fraud nor misrepresentation in the cadastral
Regional Executive Director, Region VI, Iloilo City); The CA cited27 the case of SAAD Agro-Industries, Inc. v. proceedings and in the issuance of the title in Espinosa's favor.
Republic of the Philippines. 28 The argument for the State is merely that the property was
4. Declaring that defendant Leonila Caliston has better right unlawfully included in the certificate of title because it is of the
over the subject lot as against intervenors Spouses Dioscoro In a Resolution29 dated February 4, 2009, the CA denied the public domain.
and Estrella Escarda; and State's Motion for Reconsideration.
Since the case is one for reversion and not one for land
5. Ordering the intervenors to pay defendant Leonila Caliston Hence, this petition. registration, the burden is on the State to prove that the
the following sums: property was classified as timberland or forest land at the time
The lone issue presented is whether the State has sufficiently it was decreed to Espinosa.37 To reiterate, there is no burden
a) Not less than P20,000.00 for moral damages; proved that the property is part of inalienable forest land at the on Caliston to prove that the property in question is alienable
time Espinosa was granted the cadastral decree and issued a and disposable land. 38 At this stage, it is reasonable to
b) Not less than Pl0,000.00 for exemplary damages; title. presume that Espinosa, from whom Caliston derived her title,
had already established that the property is alienable and
c) Not less than Pl0,000.00 for attorney's fees, plus so much We deny the petition. disposable land considering that she succeeded in obtaining
appearance fees of ₱2,000.00 incurred and/or paid by the OCT over it.39 In this reversion proceeding, the State must
answering defendant in connection with this case; and I prove that there was an oversight or mistake in the inclusion of
the property in Espinosa' s title because it was of public
d) Not less than ₱5,000.00 for expenses of litigation. The State failed to prove that the property was classified as dominion. This is consistent with the rule that the burden of
forest land at the time of the grant of the cadastral decree and proof rests on the party who, as determined by the pleadings
SO ORDERED. 18 issuance of title to Espinosa. or the nature of the case, asserts the affirmative of an issue.40
18

Here, the State hinges its whole claim on its lone piece of Director of Public Works and the Director of Forestry, among can hardly be considered part of the timberland or forest
evidence, the land classification map prepared in 1986. others, was published in the Official Gazette and that Governor reserve. L.C. Map No. 2961, which purports to be the "correct
General Smith's Proclamation of 1908 itself recognizes private map of the areas demarcated as permanent forest pursuant of
The records show, however, that LC Map No. 2978 was not rights.48 the provisions of P.D. No. 705 as amended" was made only in
formally offered in evidence. The rules require that 1980. Thus, the delineation of the areas was made nine (9)
documentary evidence must be formally offered in evidence We stress that our ruling is not inconsistent with the doctrine years after Orcullo was awarded the free patent over the
after the presentation of testimonial evidence, and it may be that forest lands are outside the commerce of man and subject lot.
done orally, or if allowed by the court, in writing.41 Due unsusceptible of private appropriation.1âwphi1 Neither are we
process requires a formal offer of evidence for the benefit of changing the rule on imprescriptibility of actions for reversion. xxx
the adverse party, the trial court, and the appellate courts.42 We are merely deciding on the facts as proved by the record.
This gives the adverse party the opportunity to examine and To allow a reversion based on a classification made at the time Obviously, private interests have intervened before
oppose the admissibility of the evidence.43 When evidence when the property was already declared private property by classification was made pursuant to P.D. No. 705. Not only has
has not been formally offered, it should not be considered by virtue of a decree would be akin to expropriation of land Orcullo by herself and through her predecessors-ininterest
the court in arriving at its decision.44 Not having been offered without due process of law.49 cultivated and possessed the subject lot since 1930, a free
formally, it was error for the trial court to have considered the patent was also awarded to her and a title issued in her name
survey map. Consequently, it also erred in ordering the At this juncture, we agree with the CA' s application of SAAD as early as 1971. In fact, it appears that the issuance of the
reversion of the property to the mass of the public domain on AgroIndustries, Inc., 50 which involved a complaint for free patent and certificate of title was regular and in order.
the basis of the same. annulment of title and reversion of a lot covered by a free Orcullo complied with the requisites for the acquisition of free
patent and original title. To support its claim that the lot was patent provided under Commonwealth Act No. 141 (Public
Moreover, even assuming that the survey can be admitted in part of the timberland and forest reserve, the State submitted Land Act), as certified by the Director of Lands and approved
evidence, this will not help to further the State's cause.1âwphi1 as evidence a photocopy of a land classification map. This by the Secretary of Agriculture and Natural Resources.
This is because the only fact proved by the map is one already map also became the basis of the testimonies of City
admitted by the State, that is, that the land was reclassified in Environment and Natural Resources Office officers declaring xxx
1986.45 This fact does not address the that the lot falls within the timberland or forest reserve. The
presumption/conclusion that Espinosa has, at the time of the State, however, failed to submit either a certified true copy or The Regalian doctrine is well-enshrined not only in the present
cadastral proceedings conducted in 1955, proved that the land an official publication of the map, prompting the trial court to Constitution, but also in the 1935 and 1973 Constitutions. The
is alienable and disposable, as evidenced by the decree deny its admission in evidence. After proceedings, the trial Court has always recognized and upheld the Regalian doctrine
issued in his favor in 1962. court dismissed the complaint due to the State's failure to as the basic foundation of the State's property regime.
show that the subject lot therein is part of the timberland or Nevertheless, in applying this doctrine, we must not lose sight
II forest reserve or has been classified as such before the of the fact that in every claim or right by the Government
issuance of the free patent and the original title. The CA, against one of its citizens, the paramount considerations of
The reclassification of the area where the property is located in relying on the map, reversed the trial court. fairness and due process must be observed. Respondent in
1986 should not prejudice Espinosa and her successor-in- this case failed to show that the subject lot is part of timberland
interest.46 Apropos is the case of Sta. Monica Industrial and When the case was brought before this court, we reinstated or forest reserve it adverted to. In the face of the
Dev't Corp. v. Court of Appeals. 47 In that case, the State the trial court's decision. We held that the photocopy of the uncontroverted status of Free Patent No. 473408 and OCT No.
offered in evidence a land classification map to prove that at land classification map cannot be considered in evidence 0-6667 as valid and regular issuances, respondent's insistence
the time the land was decreed to the original owner, it had not because it is excluded under the best evidence rule. We on the classification of the lot as part of the forest reserve must
yet been released and still fell within the forest zone. However, emphasized that all parties, including the Government, are be rejected. 52
the map did not conclusively state the actual classification of bound by the rules of admissibility and must comply with it-
the land at the time it was adjudicated to the original owner. The rules of admissibility must be applied uniformly. The same These principles laid down in SAAD Agro-Industries, Inc.
We thus ruled that the State failed to prove that the titles rule holds true when the Government is one of the parties. The undoubtedly apply here. As part of fair play and due process,
should be annulled- Finally, we find the need to emphasize Government, when it comes to court to litigate with one of its the State is as bound by the rules on formal offer of evidence
that in an action to annul a judgment, the burden of proving the citizens, must submit to the rules of procedure and its rights as much as every private party is. More, the State's
judgment's nullity rests upon the petitioner. The petitioner must and privileges at every stage of the proceedings are subsequent reclassification of the area where the property is
establish by clear and convincing evidence that the judgment substantially in every respect the same as those of its citizens; situated cannot be used to defeat the rights of a private citizen
is fatally defective. When the proceedings were originally filed it cannot have a superior advantage. This is so because when who acquired the land in a valid and regular proceeding
by the Republic before the Court of Appeals, the petitioner a [sovereign] submits itself to the jurisdiction of the court and conducted 24 years earlier.
contended that when the decree in favor of De Perio was participates therein, its claims and rights are justiciable by
issued by Judge Ostrand in 1912 the parcels of land were still every other principle and rule applicable to the claims and The result would have been different had the State proved that
part of the inalienable public forests. However, petitioner's rights of the private parties under similar circumstances. the property was already classified as part of forest land at the
case rested solely on land classification maps drawn several Failure to abide by the rules on admissibility renders the L.C. time of the cadastral proceedings and when title was decreed
years after the issuance of the decree in 1912. These maps fail Map submitted by respondent inadmissible as proof to show to Espinosa in 1962. However, it failed to discharge this
to conclusively establish the actual classification of the land in that the subject lot is part of the forest reserve. 51 burden; the grant of title which carries with it the presumption
1912 and the years prior to that. Before this Court, petitioner that Espinosa had already proved the alienable character of
reiterates said contention and refers, for the first time, to a We went on to explain that even if the map was admitted in the property in the cadastral proceedings stands. To grant the
1908 proclamation reserving the land in Zambales as a naval evidence to prove that the lot was classified as part of the reversion based on a subsequent reclassification, more so on
reservation and alleging that the subject parcels of land are timberland or forest reserve, the classification was made long lack of evidence, would amount to taking of private property
parts thereof. These, for reasons discussed earlier, are after private interests had intervened. Not only was the lot without just compensation and due process of law.53 This,
insufficient to overcome the legal presumption in favor of the already occupied and cultivated, a free patent and a certificate however, is not what our Constitution envisions; fairness and
decree's regularity, more so when we consider that notice of of title were also awarded and issued years ahead of the due process are paramount considerations that must still be
the application for registration and the date of hearing thereof, classification- Even assuming that the L.C. Map submitted by observed.54
addressed to the Attorney General, the Director of Lands, the respondent is admissible in evidence, still the land in question
19

WHEREFORE, the petition for review on certiorari is DENIED. concerning the validity of the certificate of title based on fraud in conformity with the law or not is a question which the
The Court of Appeals' July 25, 2008 Decision and February 4, should be raised within one year from the date of the issuance government may raise, but until it is raised by the government
2009 Resolution are AFFIRMED. No costs. of the patent. Thereafter the certificate of title based thereon and set aside, the defendant can not question it. The legality of
becomes indefeasible.... In this case the land in question is not the grant is a question between the grantee and the
SO ORDERED. a private property as the Director of Lands and the Secretary government."7 The above citation was repeated ipsissimis
of Agriculture and Natural Resources have always sustained verbis in Salazar v. Court of Appeals.8 Bereft as petitioners
A. Imperium and Dominium the public character thereof for having been formed by were of the right of ownership in accordance with the findings
reclamation.... The only remedy therefore, available to the of the Court of Appeals, they cannot, in the language of Reyes
G.R. No. L-30389 December 27, 1972 appellants is an action for reconveyance on the ground of v. Rodriguez, 9 "question the [title] legally issued." 10 The
PEDRO LEE HONG HOK, SIMEON LEE HONG HOK, fraud. In this case we do not see any fraud committed by second assignment of error is thus disposed of.
ROSITA LEE HONG HOK and LEONCIO LEE HONG HOK, defendant-appellant Aniano David in applying for the purchase
petitioners, of the land involved through his Miscellaneous Sales 2. As there are overtones indicative of skepticism, if not
vs. ANIANO DAVID, THE HON. SECRETARY OF Application No. MSA-V-26747, entered in the records of the of outright rejection, of the well-known distinction in public law
AGRICULTURE AND NATURAL RESOURCES, THE Bureau of Lands [Miscellaneous Sales] Entry No. V-9033, between the government authority possessed by the state
DIRECTOR OF LANDS and COURT OF APPEALS, because everything was done in the open. The notices which is appropriately embraced in the concept of sovereignty,
respondents. regarding the auction sale of the land were published, the and its capacity to own or acquire property, it is not
FERNANDO, J.:p actual sale and award thereof to Aniano David were not inappropriate to pursue the matter further. The former comes
clandestine but open and public official acts of an officer of the under the heading of imperium and the latter of dominium. The
Petitioners 1 in this appeal by certiorari would have us reverse Government. The application was merely a renewal of his use of this term is appropriate with reference to lands held by
a decision of respondent Court of Appeals affirming a lower deceased wife's application, and the said deceased occupied the state in its proprietary character. In such capacity, it may
court judgment dismissing their complaint to have the Torrens the land since 1938." 4 provide for the exploitation and use of lands and other natural
Title 2 of respondent Aniano David declared null and void. resources, including their disposition, except as limited by the
What makes the task for petitioners quite difficult is that their On such finding of facts, the attempt of petitioners to elicit a Constitution. Dean Pound did speak of the confusion that
factual support for their pretension to ownership of such different conclusion is likely to be attended with frustration. The existed during the medieval era between such two concepts,
disputed lot through accretion was rejected by respondent first error assigned predicated an accretion having taken place, but did note the existence of res publicae as a corollary to
Court of Appeals. Without such underpinning, they must notwithstanding its rejection by respondent Court of Appeals, dominium." 11 As far as the Philippines was concerned, there
perforce rely on a legal theory, which, to put it mildly, is would seek to disregard what was accepted by respondent was a recognition by Justice Holmes in Cariño v. Insular
distinguished by unorthodoxy and is therefore far from Court as to how the disputed lot came into being, namely by Government, 12 a case of Philippine origin, that "Spain in its
persuasive. A grant by the government through the appropriate reclamation. It does not therefore call for any further earlier decrees embodied the universal feudal theory that all
public officials 3 exercising the competence duly vested in consideration. Neither of the other two errors imputed to lands were held from the Crown...." 13 That was a
them by law is not to be set at naught on the premise, respondent Court, as to its holding that authoritative doctrines manifestation of the concept of jura regalia, 14 which was
unexpressed but implied, that land not otherwise passing into preclude a party other than the government to dispute the adopted by the present Constitution, ownership however being
private ownership may not be disposed of by the state. Such validity of a grant and the recognition of the indefeasible vested in the state as such rather than the head thereof. What
an assumption is at war with settled principles of constitutional character of a public land patent after one year, is possessed was stated by Holmes served to confirm a much more
law. It cannot receive our assent. We affirm. of merit. Consequently, as set forth at the outset, there is no extensive discussion of the matter in the leading case of
justification for reversal. Valenton v. Murciano, 15 decided in 1904. One of the royal
The decision of respondent Court of Appeals following that of decrees cited was incorporated in the Recopilacion de Leyes
the lower court makes clear that there is no legal justification 1. More specifically, the shaft of criticism was let loose de las Indias 16 in these words: "We having acquired full
for nullifying the right of respondent Aniano David to the by petitioner aimed at this legal proposition set forth in the sovereignty over the Indies and all lands, territories, and
disputed lot arising from the grant made in his favor by exhaustive opinion of then Justice Salvador Esguerra of the possessions not heretofore ceded away by our royal
respondent officials. As noted in the decision under review, he Court of Appeals, now a member of this Court: "There is, predecessors, or by us, or in our name, still pertaining to the
"acquired lawful title thereby pursuant to his miscellaneous furthermore, a fatal defect of parties to this action. Only the royal crown and patrimony, it is our will that all lands which are
sales application in accordance with which an order of award Government, represented by the Director of Lands, or the held without proper and true deeds of grant be restored to us
and for issuance of a sales patent was made by the Director of Secretary of Agriculture and Natural Resources, can bring an according as they belong to us, in order that after reserving
Lands on June 18, 1958, covering Lot 2892 containing an area action to cancel a void certificate of title issued pursuant to a before all what to us or to our viceroys audiences, and
of 226 square meters, which is a portion of Lot 2863 of the void patent (Lucas vs. Durian, 102 Phil. 1157; Director of governors may seem necessary for public squares, ways,
Naga Cadastre. On the basis of the order of award of the Lands vs. Heirs of Ciriaco Carlo, G.R. No. L-12485, July 31, pastures, and commons in those places which are peopled,
Director of Lands the Undersecretary of Agriculture and 1959). This was not done by said officers but by private parties taking into consideration not only their present condition, but
Natural Resources issued on August 26, 1959, Miscellaneous like the plaintiffs, who cannot claim that the patent and title also their future and their probable increase, and after
Sales Patent No. V-1209 pursuant to which OCT No. 510 was issued for the land involved are void since they are not the distributing to the natives what may be necessary for tillage
issued by the Register of Deeds of Naga City to defendant- registered owners thereof nor had they been declared as and pasturage, confirming them in what they now have and
appellee Aniano David on October 21, 1959. According to the owners in the cadastral proceedings of Naga Cadastre after giving them more if necessary, all the rest of said lands may
Stipulation of Facts, since the filing of the sales application of claiming it as their private property. The cases cited by remain free and unencumbered for us to dispose of as we may
Aniano David and during all the proceedings in connection with appellants are not in point as they refer to private registered wish." 17
said application, up to the actual issuance of the sales patent lands or public lands over which vested rights have been
in his favor, the plaintiffs-appellants did not put up any acquired but notwithstanding such fact the Land Department It could therefore be affirmed in Montano v. Insular
opposition or adverse claim thereto. This is fatal to them subsequently granted patents to public land applicants."5 Government" 18 that "as to the unappropriated public lands
because after the registration and issuance of the certificate Petitioner ought to have known better. The above excerpt is constituting the public domain the sole power of legislation is
and duplicate certificate of title based on a public land patent, invulnerable to attack. It is a restatement of a principle that vested in Congress, ..." 19 They continue to possess that
the land covered thereby automatically comes under the dates back to Maninang v. Consolacion, 6 a 1908 decision. As character until severed therefrom by state grant. 20 Where, as
operation of Republic Act 496 subject to all the safeguards was there categorically stated: "The fact that the grant was in this case, it was found by the Court of Appeals that the
provided therein.... Under Section 38 of Act 496 any question made by the government is undisputed. Whether the grant was disputed lot was the result of reclamation, its being correctly
20

categorized as public land is undeniable. 21 What was held in WHEREFORE, the decision of respondent Court of Appeals of B'LAAN TRIBAL FARMER'S ASSOCIATION, INTER-
Heirs of Datu Pendatun v. Director of Lands 22 finds January 31, 1969 and its resolution of March 14, 1969 are PEOPLE'S EXCHANGE, INC. and GREEN FORUM-
application. Thus: "There being no evidence whatever that the affirmed. With costs against petitioners-appellants. WESTERN VISAYAS, intervenors.
property in question was ever acquired by the applicants or COMMISSION ON HUMAN RIGHTS, intervenor.
their ancestors either by composition title from the Spanish B. IPRA Law IKALAHAN INDIGENOUS PEOPLE and HARIBON
Government or by possessory information title or by any other FOUNDATION FOR THE CONSERVATION OF NATURAL
means for the acquisition of public lands, the property must be G.R. No. 135385 December 6, 2000 RESOURCES, INC., intervenor.
held to be public domain." 23 For it is well-settled "that no ISAGANI CRUZ and CESAR EUROPA, petitioners, PER CURIAM:
public land can be acquired by private persons without any vs.
grant, express or implied, from the government." 24 It is SECRETARY OF ENVIRONMENT AND NATURAL Petitioners Isagani Cruz and Cesar Europa brought this suit for
indispensable then that there be a showing of a title from the RESOURCES, SECRETARY OF BUDGET AND prohibition and mandamus as citizens and taxpayers, assailing
state or any other mode of acquisition recognized by law. 25 MANAGEMENT and CHAIRMAN and COMMISSIONERS OF the constitutionality of certain provisions of Republic Act No.
The most recent restatement of the doctrine, found in an THE NATIONAL COMMISSION ON INDIGENOUS 8371 (R.A. 8371), otherwise known as the Indigenous Peoples
opinion of Justice J.B.L. Reyes, follows: 26 "The applicant, PEOPLES, respondents. Rights Act of 1997 (IPRA), and its Implementing Rules and
having failed to establish his right or title over the northern HON. JUAN M .FLAVIER, HON. PONCIANO BENNAGEN, Regulations (Implementing Rules).
portion of Lot No. 463 involved in the present controversy, and BAYANI ASCARRAGA, EDTAMI MANSAYANGAN, BASILIO
there being no showing that the same has been acquired by WANDAG, EVELYN DUNUAN, YAOM TUGAS, ALFREMO In its resolution of September 29, 1998, the Court required
any private person from the Government, either by purchase or CARPIANO, LIBERATO A. GABIN, MATERNIDAD M. respondents to comment.1 In compliance, respondents
by grant, the property is and remains part of the public COLAS, NARCISA M. DALUPINES, BAI KIRAM-CONNIE Chairperson and Commissioners of the National Commission
domain." 27 To repeat, the second assignment of error is SATURNO, BAE MLOMO-BEATRIZ T. ABASALA, DATU on Indigenous Peoples (NCIP), the government agency
devoid of merit. BALITUNGTUNG-ANTONIO D. LUMANDONG, DATU created under the IPRA to implement its provisions, filed on
MANTUMUKAW TEOFISTO SABASALES, DATU October 13, 1998 their Comment to the Petition, in which they
3. The last error assigned would take issue with this EDUAARDO BANDA, DATU JOEL UNAD, DATU RAMON defend the constitutionality of the IPRA and pray that the
portion of the opinion of Justice Esguerra: "According to the BAYAAN, TIMUAY JOSE ANOY, TIMUAY MACARIO D. petition be dismissed for lack of merit.
Stipulation of Facts, since the filing of the sales application of SALACAO, TIMUAY EDWIN B. ENDING, DATU
Aniano David and during all the proceedings in connection with SAHAMPONG MALANAW VI, DATU BEN PENDAO On October 19, 1998, respondents Secretary of the
said application, up to the actual issuance of the sales patent CABIGON, BAI NANAPNAY-LIZA SAWAY, BAY INAY Department of Environment and Natural Resources (DENR)
in his favor, the DAYA-MELINDA S. REYMUNDO, BAI TINANGHAGA and Secretary of the Department of Budget and Management
plaintiffs-appellants did not put up any opposition or adverse HELINITA T. PANGAN, DATU MAKAPUKAW ADOLINO L. (DBM) filed through the Solicitor General a consolidated
claim thereto. This is fatal to them because after the SAWAY, DATU MAUDAYAW-CRISPEN SAWAY, VICKY Comment. The Solicitor General is of the view that the IPRA is
registration and issuance of the certificate and duplicate MAKAY, LOURDES D. AMOS, GILBERT P. HOGGANG, partly unconstitutional on the ground that it grants ownership
certificate of title based on a public land patent, the land TERESA GASPAR, MANUEL S. ONALAN, MIA GRACE L. over natural resources to indigenous peoples and prays that
covered thereby automatically comes under the operation of GIRON, ROSEMARIE G. PE, BENITO CARINO, JOSEPH the petition be granted in part.
Republic Act 496 subject to all the safeguards provided therein JUDE CARANTES, LYNETTE CARANTES-VIVAL,
... Under Section 38 of Act 496 any question concerning the LANGLEY SEGUNDO, SATUR S. BUGNAY, CARLING On November 10, 1998, a group of intervenors, composed of
validity of the certificate of title based on fraud should be raised DOMULOT, ANDRES MENDIOGRIN, LEOPOLDO ABUGAN, Sen. Juan Flavier, one of the authors of the IPRA, Mr.
within one year from the date of the issuance of the patent. VIRGILIO CAYETANO, CONCHITA G. DESCAGA, LEVY Ponciano Bennagen, a member of the 1986 Constitutional
Thereafter the certificate of title based thereon becomes ESTEVES, ODETTE G. ESTEVEZ, RODOLFO C. AGUILAR, Commission, and the leaders and members of 112 groups of
indefeasible ..." 28 Petitioners cannot reconcile themselves to MAURO VALONES, PEPE H. ATONG, OFELIA T. DAVI, indigenous peoples (Flavier, et. al), filed their Motion for Leave
the view that respondent David's title is impressed with the PERFECTO B. GUINOSAO, WALTER N. TIMOL, MANUEL to Intervene. They join the NCIP in defending the
quality of indefeasibility. In thus manifesting such an attitude, T. SELEN, OSCAR DALUNHAY, RICO O. SULATAN, RAFFY constitutionality of IPRA and praying for the dismissal of the
they railed to accord deference to controlling precedents. As MALINDA, ALFREDO ABILLANOS, JESSIE ANDILAB, petition.
far back as 1919, in Aquino v. Director of MIRLANDO H. MANGKULINTAS, SAMIE SATURNO,
Lands, 29 Justice Malcolm, speaking for the Court, stated: ROMEO A. LINDAHAY, ROEL S. MANSANG-CAGAN, On March 22, 1999, the Commission on Human Rights (CHR)
"The proceedings under the Land Registration Law and under PAQUITO S. LIESES, FILIPE G. SAWAY, HERMINIA S. likewise filed a Motion to Intervene and/or to Appear as Amicus
the provisions of Chapter VI of the Public Land Law are the SAWAY, JULIUS S. SAWAY, LEONARDA SAWAY, JIMMY Curiae. The CHR asserts that IPRA is an expression of the
same in that both are against the whole world, both take the UGYUB, SALVADOR TIONGSON, VENANCIO APANG, principle of parens patriae and that the State has the
nature of judicial proceedings, and for both the decree of MADION MALID, SUKIM MALID, NENENG MALID, responsibility to protect and guarantee the rights of those who
registration issued is conclusive and final." 30 Such a view has MANGKATADONG AUGUSTO DIANO, JOSEPHINE M. are at a serious disadvantage like indigenous peoples. For this
been followed since then. 31 The latest case in point is ALBESO, MORENO MALID, MARIO MANGCAL, FELAY reason it prays that the petition be dismissed.
Cabacug v. Lao. 32 There is this revealing excerpt appearing DIAMILING, SALOME P. SARZA, FELIPE P. BAGON,
in that decision: "It is said, and with reason, that a holder of a SAMMY SALNUNGAN, ANTONIO D. EMBA, NORMA On March 23, 1999, another group, composed of the Ikalahan
land acquired under a free patent is more favorably situated MAPANSAGONOS, ROMEO SALIGA, SR., JERSON P. Indigenous People and the Haribon Foundation for the
than that of an owner of registered property. Not only does a GERADA, RENATO T. BAGON, JR., SARING MASALONG, Conservation of Natural Resources, Inc. (Haribon, et al.), filed
free patent have a force and effect of a Torrens Title, but in SOLEDAD M. GERARDA, ELIZABETH L. MENDI, a motion to Intervene with attached Comment-in-Intervention.
addition the person to whom it is granted has likewise in his MORANTE S. TIWAN, DANILO M. MALUDAO, MINORS They agree with the NCIP and Flavier, et al. that IPRA is
favor the right to repurchase within a period of five years." 33 It MARICEL MALID, represented by her father CORNELIO consistent with the Constitution and pray that the petition for
is quite apparent, therefore, that petitioners' stand is legally MALID, MARCELINO M. LADRA, represented by her father prohibition and mandamus be dismissed.
indefensible. MONICO D. LADRA, JENNYLYN MALID, represented by
her father TONY MALID, ARIEL M. EVANGELISTA, The motions for intervention of the aforesaid groups and
represented by her mother LINAY BALBUENA, EDWARD organizations were granted.
M. EMUY, SR., SUSAN BOLANIO, OND, PULA BATO
21

Oral arguments were heard on April 13, 1999. Thereafter, the "(1) sections 51 to 53 and 59 which detail the process of to control and supervise the exploration, development,
parties and intervenors filed their respective memoranda in delineation and recognition of ancestral domains and which utilization and conservation of Philippine natural resources."7
which they reiterate the arguments adduced in their earlier vest on the NCIP the sole authority to delineate ancestral
pleadings and during the hearing. domains and ancestral lands; After due deliberation on the petition, the members of the
Court voted as follows:
Petitioners assail the constitutionality of the following "(2) Section 52[i] which provides that upon certification by the
provisions of the IPRA and its Implementing Rules on the NCIP that a particular area is an ancestral domain and upon Seven (7) voted to dismiss the petition. Justice Kapunan filed
ground that they amount to an unlawful deprivation of the notification to the following officials, namely, the Secretary of an opinion, which the Chief Justice and Justices Bellosillo,
State’s ownership over lands of the public domain as well as Environment and Natural Resources, Secretary of Interior and Quisumbing, and Santiago join, sustaining the validity of the
minerals and other natural resources therein, in violation of the Local Governments, Secretary of Justice and Commissioner of challenged provisions of R.A. 8371. Justice Puno also filed a
regalian doctrine embodied in Section 2, Article XII of the the National Development Corporation, the jurisdiction of said separate opinion sustaining all challenged provisions of the law
Constitution: officials over said area terminates; with the exception of Section 1, Part II, Rule III of NCIP
Administrative Order No. 1, series of 1998, the Rules and
"(1) Section 3(a) which defines the extent and coverage of "(3) Section 63 which provides the customary law, traditions Regulations Implementing the IPRA, and Section 57 of the
ancestral domains, and Section 3(b) which, in turn, defines and practices of indigenous peoples shall be applied first with IPRA which he contends should be interpreted as dealing with
ancestral lands; respect to property rights, claims of ownership, hereditary the large-scale exploitation of natural resources and should be
succession and settlement of land disputes, and that any doubt read in conjunction with Section 2, Article XII of the 1987
"(2) Section 5, in relation to section 3(a), which provides that or ambiguity in the interpretation thereof shall be resolved in Constitution. On the other hand, Justice Mendoza voted to
ancestral domains including inalienable public lands, bodies of favor of the indigenous peoples; dismiss the petition solely on the ground that it does not raise a
water, mineral and other resources found within ancestral justiciable controversy and petitioners do not have standing to
domains are private but community property of the indigenous "(4) Section 65 which states that customary laws and practices question the constitutionality of R.A. 8371.
peoples; shall be used to resolve disputes involving indigenous peoples;
and Seven (7) other members of the Court voted to grant the
"(3) Section 6 in relation to section 3(a) and 3(b) which defines petition. Justice Panganiban filed a separate opinion
the composition of ancestral domains and ancestral lands; "(5) Section 66 which vests on the NCIP the jurisdiction over expressing the view that Sections 3 (a)(b), 5, 6, 7 (a)(b), 8, and
all claims and disputes involving rights of the indigenous related provisions of R.A. 8371 are unconstitutional. He
"(4) Section 7 which recognizes and enumerates the rights of peoples."5 reserves judgment on the constitutionality of Sections 58, 59,
the indigenous peoples over the ancestral domains; 65, and 66 of the law, which he believes must await the filing of
Finally, petitioners assail the validity of Rule VII, Part II, specific cases by those whose rights may have been violated
(5) Section 8 which recognizes and enumerates the rights of Section 1 of the NCIP Administrative Order No. 1, series of by the IPRA. Justice Vitug also filed a separate opinion
the indigenous peoples over the ancestral lands; 1998, which provides that "the administrative relationship of expressing the view that Sections 3(a), 7, and 57 of R.A. 8371
the NCIP to the Office of the President is characterized as a are unconstitutional. Justices Melo, Pardo, Buena, Gonzaga-
"(6) Section 57 which provides for priority rights of the lateral but autonomous relationship for purposes of policy and Reyes, and De Leon join in the separate opinions of Justices
indigenous peoples in the harvesting, extraction, development program coordination." They contend that said Rule infringes Panganiban and Vitug.
or exploration of minerals and other natural resources within upon the President’s power of control over executive
the areas claimed to be their ancestral domains, and the right departments under Section 17, Article VII of the Constitution.6 As the votes were equally divided (7 to 7) and the necessary
to enter into agreements with nonindigenous peoples for the majority was not obtained, the case was redeliberated upon.
development and utilization of natural resources therein for a Petitioners pray for the following: However, after redeliberation, the voting remained the same.
period not exceeding 25 years, renewable for not more than 25 Accordingly, pursuant to Rule 56, Section 7 of the Rules of
years; and "(1) A declaration that Sections 3, 5, 6, 7, 8, 52[I], 57, 58, 59, Civil Procedure, the petition is DISMISSED.
63, 65 and 66 and other related provisions of R.A. 8371 are
"(7) Section 58 which gives the indigenous peoples the unconstitutional and invalid; Attached hereto and made integral parts thereof are the
responsibility to maintain, develop, protect and conserve the separate opinions of Justices Puno, Vitug, Kapunan, Mendoza,
ancestral domains and portions thereof which are found to be "(2) The issuance of a writ of prohibition directing the and Panganiban.
necessary for critical watersheds, mangroves, wildlife Chairperson and Commissioners of the NCIP to cease and
sanctuaries, wilderness, protected areas, forest cover or desist from implementing the assailed provisions of R.A. 8371 SO ORDERED.
reforestation."2 and its Implementing Rules;
[G.R. No. L-2746. December 6, 1906. ]
Petitioners also content that, by providing for an all- "(3) The issuance of a writ of prohibition directing the Secretary MATEO CARIÑO, Petitioner-Appellant, v. THE INSULAR
encompassing definition of "ancestral domains" and "ancestral of the Department of Environment and Natural Resources to GOVERNMENT, Respondent-Appellee.
lands" which might even include private lands found within said cease and desist from implementing Department of SYLLABUS
areas, Sections 3(a) and 3(b) violate the rights of private Environment and Natural Resources Circular No. 2, series of
landowners.3 1998; 1. PUBLIC LANDS; PRESCRIPTION; STATUTE OF
LIMITATIONS. — The statute of limitations did not run against
In addition, petitioners question the provisions of the IPRA "(4) The issuance of a writ of prohibition directing the Secretary the Crown of Spain as to its public agricultural lands in these
defining the powers and jurisdiction of the NCIP and making of Budget and Management to cease and desist from Islands. (Following Valenton Et. Al. v. Murciano, and other
customary law applicable to the settlement of disputes disbursing public funds for the implementation of the assailed cases.)
involving ancestral domains and ancestral lands on the ground provisions of R.A. 8371; and
that these provisions violate the due process clause of the 2. ID.; ID.; PRESUMPTION OF GRANT. — There is in these
Constitution.4 "(5) The issuance of a writ of mandamus commanding the Islands no conclusive presumption of a grant from the
Secretary of Environment and Natural Resources to comply Government founded merely upon long possession.
These provisions are: with his duty of carrying out the State’s constitutional mandate
22

3. ID.; ID.; PUBLIC LAND ACT. — Act No. 926, section 54, 1880, the provisions of the laws relating to the grant, seen and read by witnesses, and its existence had been
paragraph 6, is not applicable to this case. adjustment, and sale of public were taken advantage of by proved by evidence sufficient, as we stated in the opinion (p.
these deeds from the Government for these lands would be to 460), to warrant ’the finding of the court below that the
WILLARD, J. : presume something which did not exist. The appellant says in complainant’s title was derived from the Republic of Mexico,
his brief (p.10):jgc:chanrobles.com.ph and was complete and perfect at the date when the United
States acquired sovereignty in the territory of New Mexico,
The appellant, on the 23d of June, 1903, by his attorney in "The Igorot, no less than the American Indian, is an aborigine, within which the land was situated. We do not question the
fact, Metcalf A. Clarke, filed a petition in the Court of Land and is equally ignorant of the forms of law and procedure correctness to the remarks made by Mr. Justice Shiras in
Registration asking that he be inscribed as the owner of a tract necessary to protect his interests."cralaw virtua1aw library regard to evidence of possession and the presumptions which
of land in the municipality of Baguio, in the province of may under certain circumstances drawn as to the existence of
Benguet, containing 146 hectares. The Government of the There is, moreover, in the case evidence that in 1894 the a grant.
Philippine Islands, appeared in the Court of Land Registration petitioner sought to obtain title from the Government in
and opposed the petition. The Government of the United accordance with the laws then in force. In 1901 he made a "We do not deny the right of the duty of a court to presume its
States that the land was part of the military reservation of contract with Metalcalf A. Clarke, by the terms of which he existence in a proper case, in order to quiet a title and to give
Baguio. Judgment was entered in the Court of Land agreed to sell the land to Clarke for 6,000 pesos when he to long continued possession the quality of a rightful
Registration in favor of the petitioner, from which judgment the obtained title thereto from the Government, and this contract possession under a legal right. We recognized and enforced
respondents appealed in accordance with the law then in force he does not say that he is the owner, but simply that he is in such a rule in the case of United States v. Chaves decided at
to the Court of First Instance of the province of Benguet. The possession thereof. The court below found that the land is now this term. in which the question is involved. We simply say in
case was therein tried de novo, and judgment was entered worth upwards of P50,000. this case that the possession was not a duration long enough
dismissing the petition. The petitioner has brought the case to justify any such inference.
here by bill of exceptions. The possession of the land has not been of such a character
as to require the presumption of a grant. No one has lived "There is no proof of any valid grant, but on the contrary the
The petitioner presented no documentary evidence of title, upon it for many years. It was never used for anything but evidence offered by the plaintiff himself and upon which the
except a possessory information obtained in 1901. By the pasturage of animals, except insignificant portions thereof, and bases the title that he asks the court to confirm, shows the
provisions of the Mortgage Law, under which this possessory since the insurrection against Spain it has apparently not been existence of a grant from a body which had no legal power to
information was obtained (art. 394), it produced only those used by the petitioner for any purpose. make it, and which, therefore, conveyed no title whatever to its
effects which the laws give to mere possession. grantee, and the evidence is, as given by the plaintiff himself,
The petitioner relies upon the case of the United States v. that it was under this grant alone that possession of the lands
The petitioner not having shown any title from the Government, Chaves (159 U.S., 452) and the case of The United States v. was taken. We can not presume (within the time involved in
and the land being agricultural, the case is governed by the Chaves (175 U.S., 509). In the case of Hays v. The United this case) that any other and valid grant was ever made. The
decisions of this court in the cases of Valenton Et. Al. v. States (175 U.S. 248) the court said at page 261; possession of the plaintiff and of his grantors up to the time of
Murciano 1 (2 Off. Gaz., 434); Cansino Et. Al. v. Valdez Et. Al. the treaty of Guadalupe Hidalgo, in 1848, had not been long
2 (4 Off. Gaz., 488); and Tiglao v. The Insular Government 1 (4 "But this presumption is subject to the limitation that where title enough to presume a grant. (Crispin v. United States, 168
Off. Gaz., 747). In these cases it was held that the mere is claimed from a deed which is shown to be void, it will not be U.S., 208; Hayes v. United States, 170 U.S., 637, 649, 653;
possession of land such as that in controversy in this case presumed that there was an independent grant (Smith v. Hays v. The United States, ante 248.) The possession
would give the possessor and title thereto as against the Highbee, 12 Vermont,. 113), or where surrounding subsequently existing, we can not notice. Same
Government; in other words, that the statute of limitations did circumstances are inconsistent with the theory of a grant. authorities."cralaw virtua1aw library
not run against the State in reference to its agricultural lands. (Townsend v. Downer, 32 Vermont, 183).
As we understand it, it is well settled in the United States that
The petitioner, however, insists that although the statute of "The substance of this doctrine is that lapse of time any be prescription does not run against the Government as to its
limitations as such did not run against the Government of treated as helping out the presumption of a grant, but where a public lands — in other words, that if a person desires to obtain
Spain in the Philippine Islands, yet a grant is to be conclusively void grant is shown, it affords no presumption that another title to the public lands of the United States situated within the
presumed from immemorial use and occupation. To say that valid grant was made. Nor does such presumption arise if the boundaries of the States, he must do so in the way pointed out
the presumption of a grant is presumption of law is, in our surrounding circumstances are incompatible with the existence by the law. We do not understand that a person in possession
opinion, simply to say that it amounts to a statute of limitations; of a grant. In this case under consideration we can not find any of unsurveyed public lands in the State of Minnesota, for
and for a court to hold that the statute of limitations does not evidence which justifies us in believing that a legal grant can example, whose ancestors had occupied that the land for forty
run against the Government as to its public agricultural lands, have been made, and under those circumstances we can not years, could maintain in court a claim that he was the legal
and at the same time to hold that if a person has been in consider possession since the date of the treaty as dispensing owner of the lands by reason of the presumption that the
possession of such lands for thirty years it is conclusively with the requirement that the title, if not perfect at that time, United States had granted the land to his ancestors, a
presumed that the Government has given him a deed therefor, was one which the claimant would have a lawful right to make presumption founded not upon any proceedings taken in the
would be to make two rulings directly inconsistent with each perfect had the territory not been acquired by the United General Land Office to acquire a patent thereto, but upon the
other. States."cralaw virtua1aw library mere possession for that length of time.

Considered as a presumption of fact, the contention could not In the case of Chaves v. The United States (175 U.S., 552) the The same is true of the public lands of Spain in the Philippine
be sustained in this particular case. Here the surrounding court made the following statement at page Islands. In the case of Valenton Et. Al. v. Marciano it was
circumstances are incompatible with the existence of a grant, It 562:jgc:chanrobles.com.ph said:jgc:chanrobles.com.ph
is known that for nearly three hundred years all attempts to
convert the Igorots of the Province of Benguet to the Christian "Finally, it distinctly appears that the possession of the parties "While the State has always recognized the right of the
religion completely failed, and that during that time they is insufficient in length of time to prove a valid title. In United occupant to a deed if he proves a possession for a sufficient
remained practically in the same condition as they were when States v. Chaves (159 U.S., 452) the possession was under length of time, yet it has always insisted that he must make
the Islands were first occupied by the Spaniards. To presume the claim of a grant made by the governor of New Mexico to that proof before the proper administrative officers, and obtain
as a matter of fact that during that time, and down to at least the alleged grantees. The grant had been lost, but it had been
23

from them his deed, and until he did the State remained the In the royal decree of the 13th of February, 1894, published in how far it will insist upon theoretical relations of the subject to
absolute owner."cralaw virtua1aw library the Official Gazzette of Manila of the 17th of April, 01894, it is the former sovereign and how far it will recognize actual facts.
provided in article 4 as follows:jgc:chanrobles.com.ph
But in any event, and whatever the law may be elsewhere, it Page 212 U. S. 450
seems clear that this doctrine of presumptive grant can not "ART. 4. The title to all agricultural lands which were capable
apply to the Philippines in view of the Spanish legislation for of adjustment (composicion) under the royal decree of the 25th The acquisition of the Philippines was not for the purpose of
the Indies. From time to time there were promulgated laws of June, 1880, but the adjustments of which decree in the acquiring the lands occupied by the inhabitants, and under the
which required the person in possession of public lands to Gaceta de Manila, will revert to the State. Any claim to such Organic Act of July 1, 1902, c. 1369, 32 Stat. 691, providing
exhibit their titles or grants thereto. If these titles or grants were lands by those who might have applied for the adjustment of that property rights are to be administered for the benefit of the
found to be good, they were confirmed, but if they were not, or the same, but who have not done so as the above mentioned inhabitants, one who actually owned land for many years
if the persons had no grants or titles at all, they were evicted date, will not avail them in any way or at any time."cralaw cannot be deprived of it for failure to comply with certain
from the land. For example, in Law 14, title 12, 4, virtua1aw library ceremonies prescribed either by the acts of the Philippine
Recompilation of the Laws of the Indies, it is Commission or by Spanish law.
stated:jgc:chanrobles.com.ph In view of these provisions of the law, it seems to us
impossible to say that as to the public agricultural lands in the The Organic Act of the Philippines made a bill of rights
"We therefore order and command that all viceroys and Philippines there existed a conclusive presumption after a embodying safeguards of the Constitution, and, like the
presidents of pretrial courts designate, at such times as shall to lapse of thirty or any other number of years that the Constitution, extends those safeguards to all.
them most expedient, a suitable period within which all Government of Spain had granted to the possessor thereof a
possessors of tracts, farms, plantations, and estates shall legal title thereto. Every presumption of ownership is in favor of one actually
exhibit to them and to the court officers appointed by them for occupying land for many years, and against the government
this purpose their title deeds thereto. And those who are in The plaintiff is not entitled to the benefits of paragraph 6 of which seeks to deprive him of it, for failure to comply with
possession by virtue of proper deeds and receipts or by virtue section 54 of Act No. 926, the Public Land Act, for the reason provisions of a subsequently enacted registration act.
of just prescriptive rights shall be protected, and all the rest that act is not applicable to the Province of Benguet. The
shall be restored to us to be disposed of at our will. judgment of the court below is affirmed, with the costs of this Title by prescription against the crown existed under Spanish
instance against the Appellant. law in force in the Philippine Islands prior to their acquisition by
In the Royal Cedula of October 15, 1754, it was provided — the United States, and one occupying land in the Province of
After the expiration of twenty days let judgment be entered Benguet for more than fifty years before the Treaty of Paris is
"that any and all persons who, since the year 1700, and up to accordingly and ten days thereafter the case be returned to the entitled to the continued possession thereof.
the date of promulgation and publication of said order, shall court below for execution. So ordered.
have occupied royal lands, whether or not the same shall be 7 Phil. 132 reversed.
cultivated or tenanted, may, either in person or through their Cariño v. Insular Government,
attorneys or representatives, appear and exhibit to said 212 U.S. 449 (1909) The facts are stated in the opinion.
subdelegates the titles and patents by virtue of which said Annotate this Case
lands are occupied. Said subdelegates will designate as the Syllabus | Case Page 212 U. S. 455
period within which documents must be presented a term U.S. Supreme Court
sufficient in length and proportionate to the distance the Cariño v. Insular Government, 212 U.S. 449 (1909) MR. JUSTICE HOLMES delivered the opinion of the Court.
interested party may have to travel for the purpose of making Cariño v. Insular Government of the Philippine Islands
the presentation. Said subdelegates will at the same time warn 212 U.S. 449 This was an application to the Philippine Court of Land
the parties interested that in case of the failure to present their Registration for the registration of certain land. The application
title deeds within the term designated, without a just and valid was granted by the court on March 4, 1904. An appeal was
reason therefor, they will be deprived of and evicted from their ERROR TO THE SUPREME COURT OF THE PHILIPPINE taken to the Court of First Instance of the Province of Benguet
lands, and they will be granted to others."cralaw virtua1aw ISLANDS on behalf of the government of the Philippines, and also on
library behalf of the United States, those governments having taken
Syllabus possession of the property for public and military purposes.
In the regulations of June 25, 1880, it was provided as The Court of First Instance found the facts and dismissed the
follows:jgc:chanrobles.com.ph Writ of error is the general, and appeal the exceptional, application upon grounds of law. This judgment was affirmed
method of bringing Cases to this Court. The latter method is in by the supreme court, 7 Phil. 132, and the case then was
"ART. 8. If the interested parties shall not ask an adjustment of the main confined to equity cases, and the former is proper to brought here by writ of error.
the lands whose possession they are unlawfully enjoining bring up a judgment of the Supreme Court of the Philippine
within the time of one year, or, the adjustment having been Islands affirming a judgment of the Court of Land Registration The material facts found are very few. The applicant and
granted by the authorities, they shall fail to fulfill their obligation dismissing an application for registration of land. plaintiff in error is an Igorot of the Province of Benguet, where
in connection with the compromise, by paying the proper sum the land lies. For more than fifty years before the Treaty of
into the treasury, the latter will, by virtue of the authority vested Although a province may be excepted from the operation of
in it, reassert the ownership of the Stated over the lands, and Act No. 926 of 1903 of the Philippine Commission which Page 212 U. S. 456
will, after fixing the whole thereof, proceed to sell at public provides for the registration and perfecting of new titles, one
auction that part of the same which, either because it may who actually owns property in such province is entitled to Paris, April 11, 1899, as far back as the findings go, the
have been reduced to cultivation or is not located within the registration under Act No. 496 of 1902, which applies to the plaintiff and his ancestors had held the land as owners. His
forest zone, is not deemed advisable to preserve as State whole archipelago. grandfather had lived upon it, and had maintained fences
forest reservations." 1 sufficient for the holding of cattle, according to the custom of
While, in legal theory and as against foreign nations, the country, some of the fences, it seems, having been of
sovereignty is absolute, practically it is a question of strength much earlier date. His father had cultivated parts and had used
and of varying degree, and it is for a new sovereign to decide parts for pasturing cattle, and he had used it for pasture in his
24

turn. They all had been recognized as owners by the Igorots, embodying the safeguards of the Constitution, and, like the
and he had inherited or received the land from his father in If we suppose for the moment that the government's Constitution, extends those safeguards to all. It provides that
accordance with Igorot custom. No document of title, however, contention is so far correct that the Crown of Spain in form
had issued from the Spanish Crown, and although, in 1893- asserted a title to this land at the date of the Treaty of Paris, to "no law shall be enacted in said islands which shall deprive
1894 and again in 1896-1897, he made application for one which the United States succeeded, it is not to be assumed any person of life, liberty, or property without due process of
under the royal decrees then in force, nothing seems to have without argument that the plaintiff's case is at an end. It is true law, or deny to any person therein the equal protection of the
come of it, unless, perhaps, information that lands in Benguet that Spain, in its earlier decrees, embodied the universal feudal laws."
could not be conceded until those to be occupied for a theory that all lands were held from the Crown, and perhaps
sanatorium, etc., had been designated -- a purpose that has the general attitude of conquering nations toward people not § 5. In the light of the declaration that we have quoted from §
been carried out by the Philippine government and the United recognized as entitled to the treatment accorded to those 12, it is hard to believe that the United States was ready to
States. In 1901, the plaintiff filed a petition, alleging ownership, declare in the next breath that "any person" did not embrace
under the mortgage law, and the lands were registered to him, Page 212 U. S. 458 the inhabitants of Benguet, or that it meant by "property" only
that process, however, establishing only a possessory title, it is that which had become such by ceremonies of which
said. in the same zone of civilization with themselves. It is true also presumably a large part of the inhabitants never had heard,
that, in legal theory, sovereignty is absolute, and that, as and that it proposed to treat as public land what they, by native
Before we deal with the merits, we must dispose of a technical against foreign nations, the United States may assert, as Spain custom and by long association -- one of the profoundest
point. The government has spent some energy in maintaining asserted, absolute power. But it does not follow that, as factors in human thought -- regarded as their own.
that this case should have been brought up by appeal, and not against the inhabitants of the Philippines, the United States
by writ of error. We are of opinion, however, that the mode asserts that Spain had such power. When theory is left on one It is true that, by § 14, the government of the Philippines is
adopted was right. The proceeding for registration is likened to side, sovereignty is a question of strength, and may vary in empowered to enact rules and prescribe terms for perfecting
bills in equity to quiet title, but it is different in principle. It is a degree. How far a new sovereign shall insist upon the titles to public lands where some, but not all, Spanish
proceeding in rem under a statute of the type of the Torrens theoretical relation of the subjects to the head in the past, and conditions had been fulfilled, and to issue patents to natives for
Act, such as was discussed in Tyler v. Court of Registration, how far it shall recognize actual facts, are matters for it to not more than sixteen hectares of public lands actually
175 Mass. 71. It is nearer to law than to equity, and is an decide. occupied by the native or his ancestors before August 13,
assertion of legal title; but we think it unnecessary to put it into 1898. But this section perhaps might be satisfied if confined to
either pigeon hole. A writ of error is the general method of The Province of Benguet was inhabited by a tribe that the cases where the occupation was of land admitted to be public
bringing cases to this Court, an appeal the exception, confined Solicitor General, in his argument, characterized as a savage land, and had not continued for such a length of time and
to equity in the main. There is no reason for not applying the tribe that never was brought under the civil or military under such circumstances as to give rise to the understanding
general rule to this case. Ormsby v. Webb, 134 U. S. 47, 134 government of the Spanish Crown. It seems probable, if not that the occupants were owners at that date. We hesitate to
U. S. 65; Campbell v. Porter, 162 U. S. 478; Metropolitan R. certain, that the Spanish officials would not have granted to suppose that it was intended to declare every native who had
Co. v. District of Columbia, 195 U. S. 322. anyone in that province the registration to which formerly the not a paper title a trespasser, and to set the claims of all the
plaintiff was entitled by the Spanish laws, and which would wilder tribes afloat. It is true again that there is excepted from
Page 212 U. S. 457 have made his title beyond question good. Whatever may have the provision that we have quoted as to the administration of
been the technical position of Spain, it does not follow that, in the property and rights acquired by the United States such
Another preliminary matter may as well be disposed of here. It the view of the United States, he had lost all rights and was a land and property as shall be designated by the President for
is suggested that, even if the applicant have title, he cannot mere trespasser when the present government seized his land. military or other reservations,
have it registered, because the Philippine Commission's Act The argument to that effect seems to amount to a denial of
No. 926, of 1903, excepts the Province of Benguet among native titles throughout an important part of the island of Page 212 U. S. 460
others from its operation. But that act deals with the acquisition Luzon, at least, for the want of ceremonies which the
of new titles by homestead entries, purchase, etc., and the Spaniards would not have permitted and had not the power to as this land since has been. But there still remains the
perfecting of titles begun under the Spanish law. The enforce. question what property and rights the United States asserted
applicant's claim is that he now owns the land, and is entitled itself to have acquired.
to registration under the Philippine Commission's Act No. 496, The acquisition of the Philippines was not like the settlement of
of 1902, which established a court for that purpose with the white race in the United States. Whatever consideration Whatever the law upon these points may be, and we mean to
jurisdiction "throughout the Philippine Archipelago," § 2, and may have been shown to the North American Indians, the go no further than the necessities of decision demand, every
authorized in general terms applications to be made by dominant purpose of the whites in America was to occupy the presumption is and ought to be against the government in a
persons claiming to own the legal estate in fee simple, as the land. It is obvious that, however stated, the reason for our case like the present. It might, perhaps, be proper and
applicant does. He is entitled to registration if his claim of taking over the Philippines was different. No one, we suppose, sufficient to say that when, as far back as testimony or memory
ownership can be maintained. would deny that, so far as consistent with paramount goes, the land has been held by individuals under a claim of
necessities, our first object in the internal administration of the private ownership, it will be presumed to have been held in the
We come, then, to the question on which the case was islands is to do justice to the natives, not to exploit their same way from before the Spanish conquest, and never to
decided below -- namely, whether the plaintiff owns the land. country for private gain. By the Organic Act of July 1, 1902, c. have been public land. Certainly, in a case like this, if there is
The position of the government, shortly stated, is that Spain 1369, § 12, 32 Stat. 691, all the property and rights acquired doubt or ambiguity in the Spanish law, we ought to give the
assumed, asserted, and had title to all the land in the there by the applicant the benefit of the doubt. Whether justice to the
Philippines except so far as it saw fit to permit private titles to natives and the import of the organic act ought not to carry us
be acquired; that there was no prescription against the Crown, Page 212 U. S. 459 beyond a subtle examination of ancient texts, or perhaps even
and that, if there was, a decree of June 25, 1880, required beyond the attitude of Spanish law, humane though it was, it is
registration within a limited time to make the title good; that the United States are to be administered "for the benefit of the unnecessary to decide. If, in a tacit way, it was assumed that
plaintiff's land was not registered, and therefore became, if it inhabitants thereof." It is reasonable to suppose that the the wild tribes of the Philippines were to be dealt with as the
was not always, public land; that the United States succeeded attitude thus assumed by the United States with regard to what power and inclination of the conqueror might dictate, Congress
to the title of Spain, and so that the plaintiff has no rights that was unquestionably its own is also its attitude in deciding what has not yet sanctioned the same course as the proper one "for
the Philippine government is bound to respect. it will claim for its own. The same statute made a bill of rights, the benefit of the inhabitants thereof."
25

If the applicant's case is to be tried by the law of Spain, we do articles [the articles recognizing prescription of twenty and belief of those among whom he lived, was his property,
not discover such clear proof that it was bad by that law as to thirty years] may legalize their possession, and thereby acquire through a refined interpretation of an almost forgotten law of
satisfy us that he does not own the land. To begin with, the the full ownership of the said lands, by means of adjustment Spain.
older decrees and laws cited by the counsel for the plaintiff in proceedings, to be conducted in the following manner."
error seem to indicate pretty clearly that the natives were Judgment reversed.
recognized as owning some lands, irrespective of any royal This seems, by its very terms, not to apply to those declared
grant. In other words, Spain did not assume to convert all the already to be owners by lapse of time. Article 8 provides for the C. Reservations of lands of public domain are valid
native inhabitants of the Philippines into trespassers, or even case of parties not asking an adjustment of the lands of which assertions of Regalian Right
into tenants at will. For instance, Book 4, Title 12, Law 14 of they are unlawfully enjoying the possession, within one year,
the Recopilacion de Leyes de las Indias, cited for a contrary and threatens that the treasury "will reassert the ownership of G.R. No. 81564 April 26, 1990
conclusion in Valenton v. Murciano, 3 Phil. 537, while it the state over the lands," and will sell at auction such part as it
commands viceroys and others, when it seems proper, to call does not reserve. The applicant's possession was not ACTING REGISTRARS OF LAND TITLES AND DEEDS OF
for the exhibition of grants, directs them to confirm those who unlawful, and no attempt at any such proceedings against him PASAY CITY, PASIG AND MAKATI, METRO MANILA,
hold by good grants or justa prescripcion. It is true that it or his father ever was made. Finally, it should be noted that the petitioners,
natural construction of the decree is confirmed by the report of vs.
Page 212 U. S. 461 the council of state. That report puts forward as a reason for THE REGIONAL TRIAL COURT, BRANCH 57, IN MAKATI,
the regulations that, in view of the condition of almost all METRO MANILA PRESIDED OVER BY THE HONORABLE
begins by the characteristic assertion of feudal overlordship property in the Philippines, it is important to fix its status by JUDGE FRANCISCO X. VELEZ, AND THE INTESTATE
and the origin of all titles in the King or his predecessors. That general rules on the principle that the lapse of a fixed period ESTATE OF THE LATE DELFIN CASAL, represented by
was theory and discourse. The fact was that titles were legalizes completely all possession, recommends in two DOMINGO C. PALOMARES, ADMINISTRATOR, respondents.
admitted to exist that owed nothing to the powers of Spain articles twenty and thirty years, as adopted in the decree, and
beyond this recognition in their books. then suggests that interested parties not included in those G.R. No. 90176 April 26, 1990
articles may legalize their possession and acquire ownership
Prescription is mentioned again in the royal cedula of October by adjustment at a certain price. THE INTESTATE ESTATE OF THE LATE DELFIN CASAL,
15, 1754, cited in 3 Phil. 546: represented by DOMINGO C. PALOMARES,
It is true that the language of Articles 4 and 5 attributes title to ADMINISTRATOR, petitioner,
"Where such possessors shall not be able to produce title those "who may prove" possession for the necessary time, and vs.
deeds, it shall be sufficient if they shall show that ancient we do not overlook the argument that this means may prove in HONORABLE CONRADO VASQUEZ, JR., Presiding Judge,
possession, as a valid title by prescription." registration proceedings. It may be that an English BRANCH 118, RTC, RICARDO P. SANTIAGO, ET AL.,
conveyancer would have recommended an application under respondents.
It may be that this means possession from before 1700; but, at the foregoing decree, but certainly it was not calculated to
all events, the principle is admitted. As prescription, even convey to the mind of an Igorot chief the notion that ancient Tañada, Vivo & Tan for the Intestate Estate of the Late Delfin
against Crown lands, was recognized by the laws of Spain, we family possessions were in danger, if he had read every word Casal.
see no sufficient reason for hesitating to admit that it was of it. The words "may prove" (acrediten), as well, or better, in Antonio J. Dalangpan for himself and the heirs of Delfin Casal.
recognized in the Philippines in regard to lands over which view of the other provisions, might be taken to mean when Pedro S. Ravelo for Gerardo Casal.
Spain had only a paper sovereignty. called upon to do so in any litigation. There are indications that Filomeno Peralta, Jr. for Domingo C. Palomares.
registration was expected from all, but none sufficient to show
The question comes, however, on the decree of June 25, that, for want of it, ownership actually gained would be lost.
1880, for the adjustment of royal lands wrongfully occupied by SARMIENTO, J.:
private individuals in the Philippine Islands. This begins with Page 212 U. S. 463
the usual theoretic assertion that, for private ownership, there The petitioners ** charge His Honor, Judge Francisco Velez, of
must have been a grant by competent authority; but instantly The effect of the proof, wherever made, was not to confer title, the Regional Trial Court, Branch 57, Makati, Metro Manila, with
descends to fact by providing that, for all legal effects, those but simply to establish it, as already conferred by the decree, if grave abuse of discretion in issuing an order authorizing the
who have been in possession for certain times shall be not by earlier law. The royal decree of February 13, 1894, private respondent, through Domingo Palomares, to perform
deemed owners. For cultivated land, twenty years, declaring forfeited titles that were capable of adjustment under acts of ownership over a 2,574-hectare parcel of land known
uninterrupted, is enough. For uncultivated, thirty. Art. 5. So the decree of 1880, for which adjustment had not been sought, as Hacienda de Maricaban spread out in various parts of
that, when this decree went into effect, the applicant's father should not be construed as a confiscation, but as the Makati, Pasig, Taguig, Pasay City, and Parañaque. There is no
was owner of the land by the very terms of the decree. But, it is withdrawal of a privilege. As a matter of fact, the applicant controversy as to the facts.
said, the object of this law was to require the adjustment or never was disturbed. This same decree is quoted by the Court
registration proceedings that it described, and in that way to of Land Registration for another recognition of the common law On November 5, 1985, the private respondent, Domingo
require everyone to get a document of title or lose his land. prescription of thirty years as still running against alienable Palomares, as administrator of the heirs of Delfin Casal,
That purpose may have been entertained, but it does not Crown land. commenced suit with the Regional Trial Court, Branch 132,
appear clearly to have been applicable to all. The regulations Makati, Metro Manila for declaratory relief, quieting of title,
purport to have been made "for the adjustment of royal lands It will be perceived that the rights of the applicant under the cancellation of Transfer Certificate of Title No. 192, and
wrongfully occupied by private individuals." (We follow the Spanish law present a problem not without difficulties for cancellation of entries upon Original Certificate of Title No.
translation in the government's brief.) It does not appear that courts of a different legal tradition. We have deemed it proper 291.
this land ever was royal land or wrongfully occupied. In Article on that account to notice the possible effect of the change of
6, it is provided that sovereignty and the act of Congress establishing the Palomares had earlier come to this Court (February 27, 1985)
fundamental principles now to be observed. Upon a on a similar petition, and in addition, to direct the Register of
"interested parties not included within the two preceding consideration of the whole case, we are of opinion that law and Deeds to issue a duplicate owner's copy of Original Certificate
justice require that the applicant should be granted what he of Title No. 291, embracing allegedly Hacienda de Maricaban,
Page 212 U. S. 462 seeks, and should not be deprived of what, by the practice and
26

in lieu of the (alleged) lost one. On September 9, 1985, the No other opposition having been registered, this Court hereby C. Whether or not respondent Court has acquired jurisdiction
Court denied the petition for lack of merit. (G.R. No. 69834). resolves to grant the plaintiffs' prayer in the OMNIBUS to hear and decide this action;
MOTION in order to safeguard the integrity of the land
On December 19, 1985, the petitioners filed their answer. embraced in OCT 291, hereby authorizing for this purpose the D. Whether of not respondent Court committed grave abuse of
plaintiff Domingo C. Palomares: discretion amounting to lack of jurisdiction in not dismissing
On June 2, 1986, the private respondent filed a motion to this action or allowing petitioners to appeal from the orders in
admit amended complaint impleading the Republic of the 1. To order such subdivision and/or individual survey or question. 5
Philippines and the Registers of Deeds of Pasig, Makati, and surveys within Parcel II, Parcel III and Parcel IV under Survey
Pasay City as parties-respondents, and alleging, among other Plan Psu-2031 by a licensed geodetic engineer or engineers at In their comment, the private respondent averred, among other
things, that: (1) on October 1, 1906, the Court of Land plaintiffs' expense in order to facilitate and simplify the efficient things, that: (1) the respondent court, contrary to the
Registration (James Ostrand, Presiding Judge) confirmed the administration of the property described in OCT 291; and petitioners' claim, did not decide the case "before trial"; (2)
title of Dolores Pascual Casal y Ochoa, a native of Madrid, OCT No. 291 had not been validly cancelled and that the
Spain, over the 2,574-hectare parcel above-mentioned; (2) on 2. To sell, exchange, lease or otherwise dispose (of) any area rubber stamp impression thereon, "CANCELLED" is a forgery;
October 17, 1906, the Register of Deeds of Rizal issued OCT or areas or portion or portions thereof, subject to the approval (3) the act of Judge Pineda, in denying issuance of OCT No.
No. 291 in her name; (3) upon her death, and successive of the Intestate Estate Court, to cover expenses for the 291, duplicate owner's copy, can not be considered res
deaths of her heirs, the property devolved on Gerardo, payment of taxes to which the property is subject, as well as judicata because that case involved purportedly a mere
Reynaldo, Lolita, and Erlinda, all surnamed Casal, great expenses of administration and for the protection of the petition for issuance of duplicate owner's copy; (4) non-joinder
grandchildren of Dolores; (4) no conveyances or dispositions integrity of the said lands. of proper parties is not a jurisdictional defect; (5) the TCTs
of any kind have been allegedly made upon the parcel; (5) issued thereafter are a nullity because OCT No. 291 had not
TCT No. 192, which covers the same landholding, is allegedly SO ORDERED. 1 been shown to have been duly cancelled; (6) OCT No. 291 has
spurious and inexistent; (6) the State itself, by placing become imprescriptible; and (7) the private respondent has a
27,213,255 square meters thereof under a military reservation Eleven days later, or on October 23, 1987 to be precise, it valid right of dominion over the property.
(Fort McKinley now Fort Bonifacio), by Proclamation No. 423, issued another order, as follows:
and fifty hectares thereof pursuant to Proclamation No. 192, In the meantime, the private respondent came to this Court on
had been guilty of landgrabbing; (7) any and all holders of any Acting on the plaintiffs MOTION dated October 15, 1987 certiorari (G.R. No. 90176) alleging that on December 15,
and all TCTs emanating therefrom or from TCT No. 192, are praying for the issuance of a Writ of Execution implementing 1987, in connection with Sp. Proc. No. P-2993 of the Regional
null, void, and of no force and effect; and (8) as a the Order of this Court dated October 12, 1987 before the Trial Court, Branch 118, Pasay City, entitled "In the matter of
consequence thereof, the heirs of Dolores Casal suffered expiration of the time to appeal, and after inquiring from the the Intestate Estate of the Late Fortunato Santiago and
various damages and attorney's fees. plaintiff's counsel for their reason in seeking the same, the Mariano Pantanilla Crisanta P. Santiago, et al., Petitioners,"
Court hereby issues this clarificatory order affirming the power Judge Conrado Vasquez, Jr. issued an order disposing of
On June 26, 1986, the petitioners filed an answer, stating, of the plaintiff Domingo C. Palomares to execute and perform certain parcels which the private respondent claims as forming
among other things, that: (1) the estate of Dolores Casal (or the acts authorized in the said Order of October 12, 1987 part and parcel of Hacienda de Maricaban.
Delfin Casal, her grandchild) is not a juridical person without the need of a Writ of Execution, where no relief has
authorized by law to bring suit; (2) the Registers of Deeds of been sought therefrom by any party, said Order being On June 20, 1988, the respondent judge in G.R. No. 81564
Makati, Pasig, and Pasay City are not the real parties in implementable at the instance of the said plaintiff Domingo C. filed his own comment, asserting, among other things, that: (1)
interest, but rather, the registered owners over which the court Palomares, anytime when the said Order becomes final 15 what he had sought to bar, by virtue of injunction, was
had not acquired jurisdiction; (3) the non-joinder of the real days after the said plaintiff received copy of the same (see incursions and forcible entries of trespassers and squatters; (2)
parties in interest is fatal; (4) OCT No. 291 has long been Section 39, Chapter IV, B.P. Blg. 129). Plaintiff Domingo C. the petitioners can not rightly claim that he had prematurely
cancelled; (5) Judge Gregorio Pineda of the then Court of First Palomares may therefore take whatever steps he considers adjudicated the case, because there was allegedly no decision
Instance of Rizal, Branch XXI, Pasig, had earlier denied appropriate for the implementation of the said Order without to begin with; (3) that he issued the writ of preliminary
prayers for the issuance of duplicate owner's copy of OCT No. need of further Orders or additional authority from this Court. injunction in order only to maintain the status quo ante bellum
291 because the land embraced therein had been validly that is, to re-place the private respondent, which had been
delivered to the Government; (6) the Supreme Court itself had SO ORDERED. 2 allegedly in prior possession, in possession; (4) he did not
denied the Casals' appeal; *** (7) as a consequence, res allegedly authorize unbridled "acts of ownership" to be
judicata is a bar; (8) prescription has also set in; and (9) the The petitioners filed a notice of appeal; the respondent court, exercised on the property; (5) all rights of dominion given
Casal's claims can not validly override the titles of innocent however, denied it" 3 "it being directed against . . . an thereon were subject to the approval of the intestate estate
purchasers for value. interlocutory order. . . 4 court; (6) he denied the notice of appeal because the order
dated October 12, 1987, was interlocutory in nature from which
On August 29, 1986, the respondent judge issued a temporary Hence, this recourse. no appeal lies; (7) as to jurisdiction, the various motions filed
restraining order, directing the petitioners to cease and desist by petitioners, allegedly accepting the court's jurisdiction, have
from performing the acts complained of. The petitioners interpose the following questions: clothed the court with jurisdiction, and that besides, the
jurisdictional question was never raised except now.
In a subsequent memorandum, the petitioners alleged that A. Whether or not respondent Court can validly decide before
Dolores Casal had conveyed the property to the Government trial in favor of private respondent the ownership and On July 7, 1988, the petitioners filed a reply traversing the
of the United States in 1906 and the Manila Railroad Company possession of the 25,743,514 square meters (of) land known respondent judge's allegations.
on which Judge Ostrand, the Presiding Judge of the Court of as "Hacienda de Maricaban", which is the main issue in this
Land Registration, later Justice of this Court, had stamped his case; On August 26, 1988, the respondent judge filed a
imprimatur. supplemental comment. He reiterated that the writ of injunction
B. Whether or not respondent Court can validly allow private was directed only on such spaces not occupied by the
On October 12, 1987, the respondent court issued an order in respondent to exercise and perform all acts of ownership and Government (Fort Bonifacio, Libingan ng mga Bayani, Ninoy
the tenor, as follows: possession over the said land before trial Aquino International Airport, Nayong Pilipino, Population
27

Commission, National Science and Development Board, and do not have the slightest doubt that they stand on Government The principle of res judicata is also a bar to the instant
National Housing Authority). property by sheer presumption that, unless otherwise shown, proceedings. It should be noted that in G.R. No. 69834, Mr.
what the Government occupies is what the Government owns. Domingo Palomares prayed:
Meanwhile, Atty. Antonio J. Dalangpan for and on behalf
purportedly of the "Heirs of Delfin Casal" and the private While there is no presumption that property is Government WHEREFORE, premises considered it is most respectfully
respondent, Domingo Palomares, file a Comment/Opposition property until otherwise shown, because the law recognizes prayed to the most Honorable Supreme Court, that in the
in Intervention", dated December 23, 1988 asking for the private ownership, thus: name of law, justice and fair play, to prevent and frustrate
outright dismissal of the petition. "land-grabbing" by the government, decision be rendered:
Art. 425. Property of private ownership, besides the patrimonial
On December 14, 1989, the private respondent filed a property of the State, provinces, cities, and municipalities, FIRST, That a thorough review of the aforementioned
manifestation, stating, among other things, that assuming OCT consists of all property belonging to private persons, either resolution of the Intermediate Appellate Court be made;
No. 291 had been cancelled, there was still basis for the individually or collectively. 7
respondent judge to prevent landgrabbers from entering into SECOND, That after due consideration, the resolution subject
vacant portions of the state embraced thereby. we find hard evidence on record that: (1) the property covered of review be set aside based on the aforestated assignment of
by OCT No. 291 had been conveyed to the United States of error;
The Court finds the issues, quintessentially, to be: America; (2) it had been later ceded to the Republic of the
Philippines, and (3) as a consequence, OCT No. 291 was THIRD, That the Order of the Lower Court dated Jan. 19, 1977
(1) Is OCT No. 291 still valid and subsisting? cancelled upon final order of Judge Ostrand. be affirmed as the lawful and valid order;

(2) Did the respondent judge, in issuing the orders, dated Be that as it may, the private respondent in G.R. No. 81564 is FOURTH, To erase all doubts by declaring OCT No. 291 as
October 12 and October 23, 1987, commit a grave abuse of pressed hard to establish the fact that portions of the property, continuously and existing validly against the whole world;
discretion equivalent to lack or excess of jurisdiction? especially the open spaces referred to in the lower court's writ
of injunction and the private respondent's manifestation of FIFTH, Clearing OCT No. 291 of all adverse claims, since the
I. December 14, 1989, and which open spaces it claims to be herein petitioners are the true and legally declared heirs; and
outside Maricaban, are indeed outside Maricaban (or OCT
Is OCT No. 291 still valid and subsisting? 291). With respect, however, to parts thereof on which Fort SIXTH, Ordering the Register of Deeds of Pasig, Rizal to issue
Bonifacio, Libingan ng mga Bayani, Ninoy Aquino International the Owner's Duplicate Copy of OCT No. 291.
The Court takes judicial notice of the fact that the hectarage Airport, Nayong Pilipino, Population Commission National
embraced by TCT No. 192 (OCT No. 291) consists of Science and Development Board, and National Housing Petitioner-Appellant further prays for other just and equitable
Government property. Three things persuade the Court: (1) the Authority sit, the hands of the private respondent are tied. reliefs.****
decrees of Proclamations Nos. 192 and 435; (2) the
incontrovertible fact that OCT No. 291 has been duly Claims that Judge Ostrand's decree was a counterfeit is not When we therefore denied that petition, we, in effect, held that
cancelled; and (3) the division of the Court of Appeals in AC- only self-serving, it finds no support from the records. The reconstitution (of lost duplicate owner's copy) was not possible
G.R. CV No. 00293, affirming the decision of Hon. Gregorio presumptions is "that official duty has been regularly because the mother title (OCT No. 291) had been duly
Pineda, Judge of the then Court of First Instance of Rizal, performed," 8 and the burden is on the private respondent to cancelled. And when we therefore declared OCT No. 291 to
Branch XXI, in LRC (GLRO) Rec. No. 2484, Case No. R-1467 prove irregular performance. The barren insistence that Judge have been cancelled, we perished all doubts as to the invalidity
thereof, entitled "In Re: Issuance of Owner's Duplicate of Ostrands order was a forgery is not sufficient to overthrow the of Mr. Palomares' pretenses of title to Maricaban. Our
Certificate of Title No. 291," as well as our own Resolution, in presumption. To begin with, the act of forgery has been judgment was conclusive not only as to Mr. Palomares, but
G.R. No. 69834, entitled "Domingo Palomares, et al., v. seasonably disputed by the petitioners. Secondly, the Acting also as to the existing status of the property. As we have held:
Intermediate Appellate Court". Registrar of Deeds of Pasig, who supposedly certified to the
fake character of Judge Ostrand's order, has himself joined the The lower Court correctly ruled that the present action is
(a) other petitioners in opposing the reconveyance sought. barred by the final judgment rendered in the previous case of
Tuason & Co. vs. Aguila, Civil Case No. Q-4275, of the Court
Proclamation No. 192 ("RESERVING FOR THE VETERANS (b) of First Instance of Rizal. The reason is plain: if the herein
CENTER SITE PURPOSES CERTAIN PARCEL OF LAND OF appellants really had a preferential right to a conveyance of the
THE PUBLIC DOMAIN SITUATED IN THE PROVINCE OF The decision in AC-G.R. No. 00293, dismissing the private land from J.M. Tuason & Co., or if the certificate of (Torrens)
RIZAL, ISLAND OF LUZON") and Proclamation No. 423 respondent's petition for the issuance of a new owner's copy of title held by Tuason & Co. were truly void and ineffective, then
("RESERVING FOR MILITARY PURPOSES CERTAIN OCT No. 291, a dismissal affirmed by this Court in G.R. No. these facts should have been pleaded by these appellants in
PARCELS OF THE PUBLIC DOMAIN SITUATED IN THE 69834, also militates against the return of the property to the the previous case (Q-4275), since such facts, if true,
MUNICIPALITY OF PASIG, TAGUIG, AND PARAÑAQUE heirs of Delfin Casal. The Appellate Court's judgment, a constituted a defense to the claim of Tuason & Co. for
PROVINCE OF RIZAL, AND PASAY CITY") have the judgment sustained by this Court, operates as, at the very recovery of possession. If appellants failed to plead such
character of official assertions of ownership, and the least, the law of the case between the parties, that OCT No. defenses in that previous case, they are barred from litigating
presumption is that they have been issued by right of 291 has been cancelled and the land covered has been the same in any subsequent proceeding, for it is a well
sovereignty and in the exercise of the State's dominical conveyed and ceded to the National Government. The fact that established rule that as between the same parties and on the
authority. We take not only judicial notice thereof 6 but accept AC-G.R. CV No. 00293 dealt with a petition for issuance of lost same subject and cause of action, a final judgment is
the same as a valid asseveration of regalian light over owner's duplicate copy is no argument because be that as it conclusive not only on matters directly adjudicated, but also as
property. may, the private respondent can not rightfully say that the heirs to any other matter that could have been raised in relation
of Delfin Casal still have title to the land. If it can not secure a thereto. 9
With respect to the premises occupied by the Libingan ng mga new owner's copy, it can mean that they have lost title thereto.
Bayani, Ninoy Aquino International Airport, Nayong Pilipino, II
the Population Commission, National Science and (c)
Development Board, and the National Housing Authority, we
28

Did the respondent judge, in issuing the order, dated October the status quo, but allowed Mr. Domingo Palomares to transact petition, but rather, the private respondent. It is indeed
12, 1987, commit a grave abuse of discretion equivalent to on the property by near right of dominion over it. distressing to note that it is the very judge who has taken the
lack of excess of jurisdiction? cudgels for the latter, in defending its interests, when he, the
Judge Velez had therefore no reason, indeed, excuse, to deny judge, should have remained a neutral magistrate. Res ipsa
(a) the Government's notice of appeal. What is plain is the fact loquitor. 19 He must get his just deserts.
that Judge Velez was hell-bent, so to speak, in blocking the
The Court has no doubt that Judge Velez is here guilty of Government's efforts to defend what rightfully belongs to it. III
grave abuse of discretion tantamount to lack or excess of
jurisdiction to warrant certiorari. As above-stated, what he gave What has obviously been lost on the parties, Judge Velez in The Court thus closes the long-drawn tale of Hacienda de
away, by virtue of reconveyance, was property that inalienably particular, is the established principle that injunction does not Maricaban. In this connection, let trial judges be cautioned on
belongs to the Government or its successors. Worse, he gave lie "to take property out of the possession or control of one the indiscriminate disposition of our dwindling natural
away property without notice to the actual possessors, that is, party and place it into that of another." 15 In this wise it has resources to private persons. Accordingly, we grant G.R. No.
the present registered owner. It is beyond debate, as we have also been held: 81564 and dismiss G.R. No. 90176, and so also, end what has
indicated, that the land had been, since the cancellation of come down as nearly a century of uncertainty, doubt, and
OCT No. 291, parcelled out to a succession of buyers and xxx xxx xxx conflict Maricaban has left in its trail. The Court has finally
owners. In the absence of notice, it acquired no jurisdiction to spoken. Let the matter rest.
decree redelivery or reconveyance. It is well-established that It is a well established doctrine in this jurisdiction that an
owners of property over which reconveyance is asserted are injunction is not the proper remedy for the recovery of WHEREFORE:
indispensable parties, without whom no relief is available and possession of real estate and the improvements thereon, as
without whom the court can render no valid judgment. 10 well as for the ejectments therefrom of the actual occupants 1. The petition in G.R. No. 81564 is GRANTED:
who claim to have title to or material interest therein. The use
Furthermore, the present holders of the land in question are of said remedy in such cases has invariably been considered (a) The Writ of Preliminary Injunction issued by our Resolution,
innocent purchasers for value, or presumed to be so in the unjustified, in open violation of the legal presumption that the dated April 13, 1988, enjoining the respondent judge from
absence of contrary evidence, against whom reconveyance bona fide possessor of a certain piece of land and enforcing his: (i) order of October 12, 1987 and (ii) the follow-
does not lie. 11 improvements thereon, holds the same under claim of up order of October 23, 1987, is made permanent and
ownership and with a just title, and as an advanced concession
(b) of the remedy to which the claimant might be entitled. (b) Original Certificate of Title No. 291 is declared duly
(Citations omitted) 16 CANCELLED;
The respondent judge can not conceal his faults behind
arguments that he did not intend to convey the premises, but xxx xxx xxx 2. The petition in G.R. No. 90176 is DISMISSED; and
rather, to secure, allegedly, vacant portions thereof from
interlopers. First, this is not stated in his order. Second, that Injunction, moreover, is an extraordinary remedy. It lies only in 3. Judge Francisco Velez is ordered to SHOW CAUSE why he
order is clear and unequivocal that Domingo Palomares has certain cases, to wit: should not be administratively dealt with for giving away, by
the right "[t]o sell, exchange, lease or otherwise dispose of any virtue of reconveyance, property that inalienably belongs to the
area or areas or portion or portions thereof . . . " 12 Third and Sec. 3. Grounds for issuance of preliminary injunction. - A Government, without notice to the registered owner, and
last, the security of the property is the lookout of the claimants, preliminary injunction may be granted at any time after the without benefit of trial or hearing; for blocking Government
and not the court's. In case the premises the respondent commencement of the action and before judgment when it is efforts to defend what rightfully belongs to it; and for filing his
judge's injunctive writ have been directed belong to others, let established: comment of June 17, 1988 and supplemental comment of
them air their plaints. August 26, 1988 without express leave of court.
(a) That the plaintiff is entitled to the relief demanded, and the
(c) whole or part of such relief consists in restraining the Costs against the private respondent.
commission or continuance of the acts complained of, or in the
The Court is also agreed that the challenged order was issued performance of an act or acts, either for a limited period or SO ORDERED.
with no benefit of trial or hearing. The private respondent can perpetually;
not validly rely on AC-G.R. No. 00293 as the "trial or hearing"
to justify the issuance of its said order, in the first place, (b) That the commission or continuance of some act
because it is a different proceeding. But above all, the private complained of during the litigation or the non-performance
respondent itself says that AC-G.R. CV No. 00293 can not be thereof would probably work injustice to the plaintiff; or
made a basis for denying reconveyance because "the . . .
petition was merely for the issuance of a new owner's (c) That the defendant is doing, threatens, or is about to do, or
duplicate copy . . . 13 Accordingly, it can not invoke that case is procuring or suffering to be done, some act probably in
and yet, repudiate its effects. It is the height of contradiction. violation of the plaintiffs rights respecting the subject of the
action, and tending to render the judgment ineffectual. 17
(d)
xxx xxx xxx
It was also grave error for the lower court to deny the Solicitor
General's notice of appeal. The Government had all the right to The conspicuous and unusual zeal with which Judge Francisco
appeal because: (1) the order of October 12, 1987 was in the Velez now defends his acts 18 has not escaped us. His Honor
nature of a final judgment, as "final judgment" is known in law should have borne in mind that in proceedings under Rule 65
(however it is captioned), that is to say, one that "finally of the Rules, such as the present cases, the judge is included
disposes of the pending action so that nothing more can be only as a nominal party. Unless otherwise ordained by this
done with it in the trial court; 14 (2) it did not merely maintain Court, he is not called upon to answer or comment on the

También podría gustarte